Retirement Planning & Employee Benefits Review Questions Flashcards

1
Q

Which of the following people would be considered a highly compensated employee for 2019?

  1. Lucy, a one percent owner whose salary last year was $132,000.
  2. Drew, a six percent owner whose salary was $48,000 last year.
  3. Cameron, an officer, who earned $80,000 last year and is the 29th highest paid employee of 96 employees.
  4. Helen, who earned $130,000 last year and is in the top 20 percent of paid employees.

a) 1 and 4.
b) 1, 2, and 4.
c) 1, 3, and 4.
d) 1, 2, 3, and 4.

A

b) 1, 2, and 4.
- Lucy and Helen are HC due to compensation being greater than $125,000. Drew is HC because she owns more than five percent of the business. Cameron is not HC because she does not have compensation greater than $125,000.

HC EEs:
Owner EEs
- Either an owner of > 5% for current or prior year, OR
- Compensation in excess of $125,000 for 2019 for prior plan year
Non-Owner EEs
- Compensation in excess of $125,000 for 2019 for prior plan year

How well did you know this?
1
Not at all
2
3
4
5
Perfectly
2
Q

Which of the following statements concerning accrued benefits in qualified plans is (are) correct?

  1. In a defined benefit plan, the participant’s accrued benefit at any point is the participant’s present account balance. The accrual for the specific year is the amount contributed to the plan on the employee’s behalf for that year.
  2. In a defined contribution plan, the accrued benefit is the benefit earned to date, using current salary and years of service. The accrued benefit earned for the year is the additional benefit that has been earned based upon the current year’s salary and service.

a) 1 only.
b) 2 only.
c) Both 1 and 2.
d) Neither 1 nor 2.

A

d) Neither 1 nor 2.
- Neither Statement 1 nor Statement 2 is correct because the plan names have been switched. Statement 1 describes a defined CONTRIBUTION plan and Statement 2 describes a defined BENEFIT plan.

In a defined CONTRIBUTION plan , the participant’s accrued benefit at any point is the participant’s present account balance. The accrual for a specific year is the amount contributed to the plan on the employee’s behalf for that year.

In a defined BENEFIT plan, the accrued benefit is the benefit earned to date, using current salary and years of service. The accrued benefit earned for the year is the additional benefit that has been earned based upon the current year’s salary and service.

How well did you know this?
1
Not at all
2
3
4
5
Perfectly
3
Q

Snikerdy Corporation owns:
- 95% of the stock of Corporation A,
- 80% of the stock of Corporation B, and
- 75% of the stock of Corporation C.
In addition, Corporation B owns 90% of Corporation D. Unrelated persons own all unaccounted for shares. Snikerdy is the common parent of a parent-subsidiary group consisting of corporations:

a) A only.
b) A and B only.
c) A, B and D only.
d) A, B, C and D.

A

c) A, B and D only.
- In this example Snikerdy is the common parent of a parent-subsidiary group consisting of Corporation A, B and D. This is because it owns 80% or more of A and B directly. Since B owns 80% of D, it will also be considered in the controlled group.

How well did you know this?
1
Not at all
2
3
4
5
Perfectly
4
Q

If a participant’s accrued benefit from a qualified defined benefit pension plan is $2,000 per month, what is the maximum life insurance death benefit coverage that the plan can provide based on the 100-to-1 ratio test?

a) $0.
b) $2,400.
c) $200,000.
d) $240,000.

A

c) $200,000.
- A qualified pension plan is limited in the amount of term life insurance it is able to purchase with plan assets. The plan must pass either one of two tests, the 25 percent test or the 100 - to - 1 ratio test. Under the 100-to-1 ratio test, the plan can purchase $200,000 ($2,000 x 100) of term life insurance death benefit cover-age.

How well did you know this?
1
Not at all
2
3
4
5
Perfectly
5
Q

Jacob is the owner of Office Mart, Inc., and he would like to establish a qualified pension plan. Jacob would like most of the plan’s current contributions to be allocated to his account. He does not want to permit loans, nor does he want Office Mart to bear the investment risk of the plan’s assets. Jacob is 47 and earns $300,000 per year. His employees’ ages are 25, 29, and 32, and they each earn $25,000 per year. Which of the following qualified pension plans would you recommend that Jacob establish?

a) Defined benefit pension plan.
b) Cash balance pension plan.
c) Money purchase pension plan.
d) Defined benefit pension plan using permitted disparity.

A

c) Money purchase pension plan.
- Because Jacob does not want Office Mart to bear the investment risk of the plan assets, the money purchase pension plan is the option listed that would fulfill his requirements.

How well did you know this?
1
Not at all
2
3
4
5
Perfectly
6
Q

Caleb, the 100 percent owner of Caleb’s Car Wash (a sole proprietorship), would like to establish a profit sharing plan. Caleb’s Car Wash’s tax year ends July 31 to coincide with the school year. What is the latest day Caleb can establish and contribute to the plan?

a) Caleb must establish and contribute to the plan by December 31 of the year in which he would like to establish the plan.
b) Caleb must establish the plan by July 31 of the year in which he would like to have the plan and contribute by May 15 of the following year assuming he filed the appropriate extensions.
c) Caleb must establish the plan by July 31 of the year in which he would like to establish the plan and contribute by December 31.
d) Caleb must establish the plan by December 31 of the year in which he would like to establish the plan and contribute to the plan by April 15 of the following year.

A

b) Caleb must establish the plan by July 31 of the year in which he would like to have the plan and contribute by May 15 of the following year assuming he filed the appropriate extensions.
- Caleb must establish the plan by July 31 of the year in which he would like to have the plan and contribute funds by May 15 (9 1/2 months after year end) the following year assuming he filed all of the appropriate extensions.

How well did you know this?
1
Not at all
2
3
4
5
Perfectly
7
Q

Super Skate, the city’s most popular roller skating rink, has a profit sharing plan for their employees. Super Skate has the following employee information: Employee ; Age ; Length of Service
Greg ; 62 ; 14 years
Marsha ; 57 ; 14 years
Jan ; 32 ; 6 months
Peter ; 22 ; 2 years
Cindy ; 19 ; 2 years
Bobby ; 17; 6 months
Mike ; 16 ; 1 year
The plan requires the standard eligibility and the least generous graduated vesting schedule available. The plan is not top heavy. All of the following statements are correct except:
a) Peter and Cindy are 20 percent vested in their benefits.
b) Greg and Marsha became 100 percent vested when they had been employed for six years.
c) Three of the seven people are eligible to participate in the plan.
d) Mike is not eligible for the plan.

A

a) Peter and Cindy are 20 percent vested in their benefits.
- The standard vesting schedule requires individuals to be 21 years of age and have one year of service before becoming eligible for the plan. Greg, Marsha, and Peter are the only individuals that meet that criteria. The least generous vesting schedules are 3-year cliff and 2-to-6-year graduated vesting. Therefore, Peter is 20 percent vested, and Marsha and Greg became 100 percent vested in the 6th year. Mike is not eligible because he is 16 years old. Cindy is not vested because she is not eligible due to her age.

How well did you know this?
1
Not at all
2
3
4
5
Perfectly
8
Q

JMG Company has three employees: Julia, Maria, and Gary. Their compensations are $50,000, $150,000, and $200,000, respectively. JMG is considering establishing a straight 10 percent profit sharing plan or an integrated profit sharing plan using a 10 percent contribution for base compensation and 15.7 percent for excess compensation. Which of the following statements is correct?

a) If the integrated plan is selected, then the total contribution for all employees is $44,799.40
b) The effect of the integrated plan results in an increase in Maria’s contribution of $1,300.
c) If the integrated plan is selected, the base contribution for all employees is $40,000.
d) If the integrated plan is selected, Gary’s total contribution is $31,400.

A

a) If the integrated plan is selected, then the total contribution for all employees is $44,799.40

How well did you know this?
1
Not at all
2
3
4
5
Perfectly
9
Q

Logan owns Airliner, Inc. and sells 100 percent of the corporate stock (all outstanding stock) on January 1 of this year to an ESOP for $5,000,000. His adjusted basis in the stock was $2,400,000. Which of the following is correct?

  1. If Logan reinvests the $5,000,000 in qualified domestic securities within 18 months, he has a carryover basis of $2,400,000 in the qualified domestic security portfolio and no current capital gain.
  2. Logan has a long-term capital gain of $2,600,000 reduced by the 20 percent small business credit; there-fore, his gain is $2,080,000 if he does not reinvest in qualified domestic securities within 18 months.
    a) 1 only.
    b) 2 only.
    c) 1 and 2.
    d) Neither 1 nor 2.
A

d) Neither 1 nor 2.
- The $5,000,000 must be reinvested within 12 months, and there is not a 20 percent small business credit. In addition, to qualify for nonrecognition of gain treatment, the Airliner, Inc. stock must have been owned by Logan for at least three years.

How well did you know this?
1
Not at all
2
3
4
5
Perfectly
10
Q

Connor is 701⁄2 on April 1 of the current year and must receive a minimum distribution from his qualified plan. The account balance had a value of $423,598 at the end of last year. The distribution period for a 70 year old is 27.4, and for a 71 year old it is 26.5. If Connor takes a $15,000 distribution next April 1st, what is the amount of the minimum distribution tax penalty?

a) $0.
b) $230.
c) $492.
d) $985.

A

c) $492
- The required minimum distribution for Connor is $15,985 ($423,598 divided by 26.5) because he is 71 years old as of December 31 of the current year. Connor only took a distribution of $15,000, therefore, the minimum distribution penalty (50%) would apply to the $985 balance. Therefore, the minimum distribution penalty is $492 (50% of the $985).

How well did you know this?
1
Not at all
2
3
4
5
Perfectly
11
Q
Jose Sequential, age 701⁄2 in October of this year, worked for several companies over his lifetime. He has worked for the following companies (A-E) and still has the following qualified plan account balances at those companies. 
Company Jose’s Account Balance
A $250,000
B $350,000
C $150,000
D $350,000
E $200,000 
Jose is currently employed with Company E. What, if any, is his required minimum distribution for the cur-rent year from all plans? Life expectancy tables are 27.4 for age 70 and 26.5 for age 71. 
a) $0. 
b) $40,146. 
c) $41,509. 
d) $47,445
A

b) $40,146
- Jose is required to take a minimum distribution for the years in which he is 701⁄2 from each qualified plan, except from his current employer ($1,100,000  27.4 = $40,146). He can delay the payment until April 1 of next year, but the question asks for the distribution required for the current year

How well did you know this?
1
Not at all
2
3
4
5
Perfectly
12
Q
Ella, age 70 on February 2, YR4, had the following account balances in a qualified retirement plan. 12/31/YR1 $300,000 
12/31/YR2 $350,000
12/31/YR3 $500,000
12/31/YR4 $478,000
12/31/YR5 $519,000
12/31/YR6 $600,000
Assuming that Ella is retired and has never taken a distribution prior to YR5, what is the total amount of minimum distribution required in YR5? Life expectancy factors according to the uniform life table are 27.4 for a 70 year old and 26.5 for a 71 year old. 
a) $18,037. 
b) $18,248. 
c) $35,597. 
d) $36,286
A

d) $36,286
- For YR4, look back to YR3: $500,000 / 27.4 = $18,248
- For YR5, look back to YR4: $478,000 / 26.5 = $18,038
- $18,248 + $18,038 = $36,286

How well did you know this?
1
Not at all
2
3
4
5
Perfectly
13
Q

Which of the following is true regarding QDROs?

a) The court determines how the retirement plan will satisfy the QDRO (i.e., split accounts, separate interest). b) In order for a QDRO to be valid, the order must be filed on Form 2932-QDRO provided by ERISA.
c) All QDRO distributions are charged a 10% early withdrawal penalty.
d) A QDRO distribution is not considered a taxable distribution if the distribution is deposited into the recipient’s IRA or qualified plan

A

c) All QDRO distributions are charged a 10% early withdrawal penalty.
- The plan document, not the court, determines how the QDRO will be satisfied. No particular form is required for a QDRO, although some specific information is required. Form 2932-QDRO is not a real form. QDRO distributions may be subject to the 10% early withdrawal penalty if the distribution is not deposited into the recipient’s IRA or qualified plan.

How well did you know this?
1
Not at all
2
3
4
5
Perfectly
14
Q

Investment portfolio risk is generally borne by the participant/employee in all of the listed qualified plans, except:

  1. Defined benefit pension plan.
  2. Cash balance pension plan.
  3. 401(k) plan.
  4. Profit sharing plan.

a) 1 and 2.
b) 2 and 3.
c) 3 and 4.
d) 1, 3, and 4.

A

a) 1 and 2.

- In Defined benefit and Cash balance pension plans, the employer bears all of the investment risk.

How well did you know this?
1
Not at all
2
3
4
5
Perfectly
15
Q

A distress termination of a qualified retirement plan occurs when:

  1. The PBGC initiates a termination because the plan was determined to be unable to pay benefits from the plan.
  2. An employer is in financial difficulty and is unable to continue with the plan financially. Generally, this occurs when the company has filed for bankruptcy, either Chapter 7 liquidation or Chapter 11 reorganization.
  3. The employer has sufficient assets to pay all benefits vested at the time, but is distressed about it.
  4. When the PBGC notifies the employer that it wishes to change the plan due to the increasing unfunded risk.

a) 2 only.
b) 1 and 2.
c) 1, 2, and 3.
d) 1, 2, and 4.

A

a) 2 only.
- Statement 2 is the definition of a distress termination. Statement 3 is standard termination. Statement 1 describes an involuntary termination. Statement 4 is simply false.

How well did you know this?
1
Not at all
2
3
4
5
Perfectly
16
Q

Nathan, age 46, is a self-employed financial planner and has Schedule C income from self-employment of $56,000. He has failed to save for retirement until now. Therefore, he would like to make the maximum contribution to his profit sharing plan. How much can he contribute to his profit sharing plan account?

a) $9,486.
b) $10,409.
c) $11,200.
d) $14,000.

A

b) $10,409.

$56,000 schedule c net income
-3,956 (less 56,000 x .9235 x .0765)
$52,044 net self-employment income
x 0.20 (0.25 / 1.25)
$10,409
How well did you know this?
1
Not at all
2
3
4
5
Perfectly
17
Q

Which statements are correct regarding penalties associated with IRA accounts?

  1. Generally, distributions made prior to 591⁄2 are subject to the 10% premature distribution penalty.
  2. There is a 50% excise tax on a required minimum distribution not made by April 1 of the year following the year in which age 701⁄2 is attained.
    a) 1 only.
    b) 2 only.
    c) 1 and 2.
    d) Neither 1 nor 2.
A

c) 1 and 2.

- Statements 1 and 2 are correct.

How well did you know this?
1
Not at all
2
3
4
5
Perfectly
18
Q

Lauren, who was divorced in 2015 and is age 55, received taxable alimony of $50,000 in 2019. In addition, she received $1,800 in earnings from a part-time job. Lauren is not covered by a qualified plan. What was the maximum deductible IRA contribution that Lauren could have made for 2019?

a) $1,800.
b) $2,800.
c) $6,000.
d) $7,000

A

d) $7,000
- The deductible IRA contribution limit is $6,000 for 2019. The additional catch-up amount, for 50 or older, is $1,000 for 2019. Alimony counts as earned income for IRA purposes. She is not covered by a qualified plan and, therefore, is not subject to AGI phaseouts. Therefore, the total (for divorce prior to 12/31/2018 - TCJA 2017) is $7,000 for 2019.

How well did you know this?
1
Not at all
2
3
4
5
Perfectly
19
Q
Ashley (age 55) is single, divorced in 2016, and has received the following items of income this year: Pension annuity income from QDRO $21,000
Interest and dividends $5,000
Alimony $1,000
W-2 Income $1,200 
What is the most that Ashley can contribute to a Roth IRA for 2019? 
a) $1,200. 
b) $2,200. 
c) $6,000. 
d) \$\$7,000.
A

b) $2,200.
- Contributions to Roth IRAs, as well as traditional IRAs, are limited to the lesser of earned income or $6,000 for 2019. Ashley has earned income of $2,200 from the alimony (for a divorce prior to 12/31/2018) and W-2 income she received. Thus, she is limited to a contribution of $2,200. The other $26,000 of income is not earned income and, therefore, is unavailable for contributions to any IRA.
- Note: An additional catch-up contribution of $1,000 for 2019 is permitted for individuals who have attained age 50 by the close of the tax year. Her total remains at $2,200 because that is all the earned income she has.

How well did you know this?
1
Not at all
2
3
4
5
Perfectly
20
Q

Which of the following statements is/are correct regarding SEP contributions made by an employer?

  1. Contributions are subject to FICA and FUTA.
  2. Contributions are currently excludable from employee-participant’s gross income.
  3. Contributions are capped at $19,000 for 2019.
    a) 1 only.
    b) 2 only
    c) 1 and 2.
    d) 1, 2, and 3.
A

b) 2 only
- Statement 2 is the only correct response. Statements 1 and 3 are incorrect. Employer contributions to a SEP are not subject to FICA and FUTA. The 401(k) elective deferral limit and the SARSEP deductible limits are $19,000 for 2019. The SEP limit is 25% of covered compensation up to $56,000 for 2019.
- Note: The maximum compensation that may be taken into account in 2019 for purposes of SEP contributions is $280,000. Therefore, the maximum amount that can be contributed to a SEP in 2019 is $56,000 (25% x $280,000, limited to $56,000).

How well did you know this?
1
Not at all
2
3
4
5
Perfectly
21
Q

A SEP is not a qualified plan and is not subject to all of the qualified plan rules; however, it is subject to many of the same rules. Which of the following are true statements?

  1. SEPs and qualified plans have the same funding deadlines.
  2. The contribution limit for SEPs and qualified plans (defined contribution) is $56,000 for the year 2019.
  3. SEPs and qualified plans have the same ERISA protection from creditors.
  4. SEPs and qualified plans have different nondiscriminatory and top-heavy rules.
    a) 1 only.
    b) 1 and 2.
    c) 2 and 4.
    d) 1, 2, 3, and 4.
A

b) 1 and 2.
- SEPs and qualified plans can be funded as late as the due date of the return plus extensions. The maximum contribution for an individual to a SEP is $56,000 for 2019 ($280,000 maximum compensation x 25%, limited to $56,000). Thus, Statements 1 and 2 are correct. Qualified plans are protected under ERISA. IRAs and SEPs do not share this protection. Both types of plans have the same nondiscriminatory and top-heavy rules

How well did you know this?
1
Not at all
2
3
4
5
Perfectly
22
Q

Emily, age 62, single, and retired, receives a defined benefit pension annuity of $1,200 per month from Greene Corporation. She is currently working part-time for Jake’s Kitchen Design and will be paid $18,000 this year (2019). Jake’s Kitchen Design has a 401(k) plan, but Emily has made no contribution to the plan, has not been allocated any forfeitures, and Jake will not contribute this year. Can Emily contribute to a traditional IRA or a Roth IRA for the year, and what is the maximum contribution?

a) $6,000 to a traditional IRA or $6,000 to a Roth IRA.
b) $0 to a traditional IRA or $7,000 to a Roth IRA.
c) $7,000 to a traditional IRA or $0 to a Roth IRA.
d) $7,000 to a traditional IRA or $7,000 to a Roth IRA

A

d) $7,000 to a traditional IRA or $7,000 to a Roth IRA
- Emily has earned income and is 50 or older. She is not an active participant in a retirement plan and even if she were, she is below the income limits.

How well did you know this?
1
Not at all
2
3
4
5
Perfectly
23
Q

Kate, age 42, earns $300,000 annually as an employee for Austin, Inc. Her employer sponsors a SIMPLE retirement plan and matches all employee contributions made to the plan dollar-for-dollar up to 3% of covered compensation. What is the maximum contribution (employer and employee) that can be made to Kate’s SIMPLE account in 2019?

a) $13,000.
b) $21,400.
c) $22,000.
d) $56,000.

A

c) $22,000.
- The maximum total contribution is $22,000. ($13,000 maximum employee contribution for 2019 + $9,000 employer match). The maximum employee contribution for 2019 is $13,000. The employer makes matching contributions up to 3% of compensation (SIMPLE maximum). Therefore, the employer can make a contribution of up to $9,000 ($300,000 compensation x 3%). Compensation is not limited with the matching contribution, only the non-elective contribution

How well did you know this?
1
Not at all
2
3
4
5
Perfectly
24
Q

Sawyer, age 25, works for Island Horticulture. Island Horticulture adopted a SIMPLE plan 6 months ago. Sawyer made an elective deferral contribution to the plan of $8,000, and Island Horticulture made a matching contribution of $2,400. Which of the following statements is/are correct?

  1. Sawyer can withdraw his entire account balance without terminating employment.
  2. Sawyer can roll his SIMPLE IRA into his traditional IRA upon terminating employment.
  3. Sawyer will be subject to ordinary income taxes on withdrawals from the SIMPLE.
  4. Sawyer may be subject to a 25% early withdrawal penalty on amounts withdrawn from the SIMPLE.
    a) 1 and 2.
    b) 1 and 3.
    c) 2, 3, and 4.
    d) 1, 3, and 4.
A

d) 1, 3, and 4.
- Statement 1 is correct. SIMPLEs must provide 100% immediate vesting of employer contributions. The entire balance is available for withdrawal. Statement 2 is incorrect. A SIMPLE IRA cannot be rolled into a traditional IRA until the participant has been in the SIMPLE IRA for two years. Tyler has only been in the SIMPLE for 6 months. Statement 3 is correct. The entire withdrawal will be subject to ordinary income tax in the year of withdrawal. Statement 4 is correct because the early withdrawal penalty for a SIMPLE is 25% for withdrawals occurring within the first two years of participation.

How well did you know this?
1
Not at all
2
3
4
5
Perfectly
25
Q

Which of the following statements is/are correct regarding TSAs and 457 deferred compensation plans? 1. Both plans require contracts between an employer and an employee.

  1. Participation in either a TSA or a 457 plan will cause an individual to be considered an “active participant” for purposes of phasing out the deductibility of traditional IRA contributions.
  2. Both plans allow 10-year forward averaging tax treatment for lump-sum distributions.
  3. Both plans must meet minimum distribution requirements that apply to qualified plans.
    a) 1 only.
    b) 1 and 4.
    c) 2, 3, and
  4. d) 1, 2, and 4
A

b) 1 and 4.
- Statements 1 and 4 are correct. Statement 2 is incorrect because a 457 plan is a deferred compensation arrangement that will not cause a participant to be considered an “active participant.” Statement 3 is incorrect because 10-year forward averaging is not permitted from either plan.

How well did you know this?
1
Not at all
2
3
4
5
Perfectly
26
Q

Which of the following are permitted investments in a 403(b) TSA (TDA) plan?

  1. An annuity contract from an insurance company.
  2. An international gold stock mutual fund.
  3. A self-directed brokerage account consisting solely of US stocks, bonds and mutual funds.
    a) 1 only.
    b) 2 only.
    c) 1 and 2.
    d) 1, 2, and 3.
A

c) 1 and 2.
- TSA (TDA) funds can only invest in annuity contracts (Statement 1) and mutual funds (Statement 2). No self-directed brokerage accounts are permitted.

How well did you know this?
1
Not at all
2
3
4
5
Perfectly
27
Q

What is the maximum catch-up for 2019 under the 457(b) plan “Final 3-Year” rule?

a) $3,000.
b) $6,000.
c) $19,000.
d) $37,000.

A

c) $19,000.

- The final 3-year catch-up is $19,000 for 2019.

How well did you know this?
1
Not at all
2
3
4
5
Perfectly
28
Q

Which of the following is false regarding a deferred compensation plan that is funded using a rabbi trust?

  1. Participants have security against the employer’s unwillingness to pay.
  2. Rabbi trusts provide the participant with security against employer bankruptcy.
  3. Rabbi trusts provide tax deferral for participants.
  4. Rabbi trusts provide the employer with a current tax deduction.
    a) None, they are all true.
    b) 2 and 4.
    c) 1, 2, and 4.
    d) 1, 2, 3, and 4.
A

b) 2 and 4.

- Rabbi trusts do not provide security against employer bankruptcy or a current tax deduction for the employer.

How well did you know this?
1
Not at all
2
3
4
5
Perfectly
29
Q

Which of the following is true regarding employer contributions to secular trusts for employee-participants of a non-qualified deferred compensation agreement?

  1. Participants have security against an employer’s unwillingness to pay at termination.
  2. Participants have security against an employer’s bankruptcy.
  3. Secular trusts provide tax deferral for employees until distribution.
  4. Secular trusts provide employers with a current income tax deduction.
    a) 3 only.
    b) 1 and 2.
    c) 1, 2, and 4.
    d) 1, 2, 3, and 4.
A

c) 1, 2, and 4.
- Secular trusts are similar to rabbi trusts except that participants do not have a substantial risk of forfeiture and thus, do not provide the employee with tax deferral. Secular trusts provide the employer with a current income tax deduction for contributions. Secular trusts protect the participant from employer unwillingness to pay because they are funded, and they protect from bankruptcy because there is no risk of forfeiture.

How well did you know this?
1
Not at all
2
3
4
5
Perfectly
30
Q

Roger receives stock options (ISOs) with an exercise price of $18 when the stock is trading at $18. Roger exercises these options two years after the date of the grant when the stock price is $39 per share. Which of the following statements is correct?

a) Upon exercise Roger will have no regular income for tax purposes.
b) Roger will have W-2 income of $21 per share upon exercise.
c) Roger will have $18 of AMT income upon exercise.
d) Roger’s adjusted basis for regular income tax will be $39 at exercise.

A

a) Upon exercise Roger will have no regular income for tax purposes.
- Roger does not have income at the date of exercise. Roger’s adjusted basis will be $18. The AMT will be the difference between the fair market value and the exercise price ($39 - $18 = $21).

How well did you know this?
1
Not at all
2
3
4
5
Perfectly
31
Q

Which of the following are characteristics of a phantom stock plan?

  1. Benefits are paid in cash.
  2. There is no equity dilution from additional shares being issued.
    a) 1 only.
    b) 2 only.
    c) 1 and 2.
    d) Neither 1 nor 2.
A

c) 1 and 2.
- The employee does not actually receive stock in a phantom plan. Instead, the employee receives credits for the stock and the benefits are later paid in cash.

How well did you know this?
1
Not at all
2
3
4
5
Perfectly
32
Q

ABC has an Employee Stock Purchase Plan (ESPP). Which statements regarding an ESPP are correct?

  1. The price may be as low as 85% of the stock value.
  2. When an employee sells stock at a gain in a qualifying disposition, all of the gain will be capital gain.
  3. There is an annual limit of $25,000 per employee.
    a) 1 only.
    b) 1 and 2.
    c) 1 and 3.
    d) 2 and 3.
A

c) 1 and 3.

- Statement 2 is incorrect because only the gain in excess of the W-2 income will be capital gain.

How well did you know this?
1
Not at all
2
3
4
5
Perfectly
33
Q

Devon was awarded 1,000 shares of restricted stock of B Corp at a time when the stock price was $14. Assume Devon properly makes an 83(b) election at the date of the award. The stock vests 2 years later at a price of $12 and Devon sells it then. What are Devon’s tax consequences in the year of sale?

a) Devon has W-2 income of $12,000.
b) Devon has a long-term capital loss of $2,000.
c) Devon has W-2 income of $14,000.
d) Devon has a $12,000 long-term capital gain.

A

b) Devon has a long-term capital loss of $2,000.
- In the year of sale, Devon will have a long-term capital loss of $2,000 ($14,000 - $12,000) because his right to the stock vested. Losses are permitted when 83(b) is elected after the right to the stock has vested.

How well did you know this?
1
Not at all
2
3
4
5
Perfectly
34
Q

Marni received 1,000 SARs at $22, the current trading price of Clippers, Inc., her employer. If Marni exercises the SARs three years after the grant and Clipper’s stock is $34 per share, which of the following statements is true?

a) Marni will have an adjusted basis of $22,000 in the Clippers, Inc. stock.
b) Marni will have W-2 income equal to $12,000.
c) Marni will have long-term capital gain of $12,000.
d) Marni will have ordinary income equal to $22,000.

A

b) Marni will have W-2 income equal to $12,000.
- At the exercise of a SAR, the employee receives the difference between the fair market value and the exercise price as W-2 income. Thus, Marni has W-2 income equal to $12,000 [($34 - $22) x 1,000].

How well did you know this?
1
Not at all
2
3
4
5
Perfectly
35
Q

Coldstone Company allows a 25% discount to all nonofficer employees. Officers are allowed a 30% dis-count on company products. Coldstone’s gross profit percent is 35%. Which of the following is true?

a) An officer who takes a 30% discount must include the extra 5% (30%-25%) in his gross income.
b) Any discounts taken by any employee is includable in the employee’s gross income because the plan is discriminatory.
c) All discounts taken by officers (30%) are includable in their gross income because the plan is discriminatory.
d) None of the discounts taken by any employee are includable in their gross income because the discount, in all cases, is less than the company’s gross profit percentage.

A

c) All discounts taken by officers (30%) are includable in their gross income because the plan is discriminatory.
- The plan is discriminatory to non-highly compensated employees; therefore, all discounts actually taken by officers are includable in the officers’ income, not just the excess of what is available to the nonofficers. Any discount taken by a nonofficer would be excluded from the employee’s gross income.

How well did you know this?
1
Not at all
2
3
4
5
Perfectly
36
Q

Juliet is married to Jack and they have one child Angela, age 14, who is in the 6th grade. Angela is a difficult child and she is cared for in the afternoon by the Sisters of Reformation, a group of Catholic nuns. Juliet pays $6,000 per year for the child care. Juliet’s company has a dependent care assistance program. If Juliet makes the maximum use of the dependent care assistance program, how much can she exclude from her income if she files a joint return with Jack?

a) $0.
b) $2,500.
c) $5,000.
d) $6,000.

A

a) $0.

- Angela is over 13 years old and, therefore, does not qualify for the dependent care assistance program.

How well did you know this?
1
Not at all
2
3
4
5
Perfectly
37
Q

Which of the following is true regarding qualified incentive stock options?

I. No taxable income will be recognized by the employee when the qualified option is granted or exercised.
II. The income from sale of the qualified option will always be taxed as capital gains when the stock is sold.
III. The income from sale of the qualified option will be taxed as ordinary income regardless of when the stock is sold.
IV. The employer will not be able to deduct the bargain element of the option as an expense under any circumstance.
V. For favorable tax treatment the option must be held two years and the stock for one year after exercise.

II and IV only.
I and V only.
III only.
I, II and V only.

A

I and V only.
- In Statement “II,” be careful of “always”! In Statement “III,” if held longer than one year, they receive capital gains treatment. In Statement “IV,” under most circumstances, the bargain element is deductible. There are exceptions when certain qualifications have not been met for deductibility, such as time employed, time to exercise in excess of rules, etc.

How well did you know this?
1
Not at all
2
3
4
5
Perfectly
38
Q

Eric works for Carpets, Inc. Carpets, Inc issued him both ISOs and NQSOs during the current year. Which of the following would be the most compelling reason why they might issue both ISOs and NQSOs?

They want to issue over $80,000 in options that are exercisable in the same year.
The NQSOs and ISOs are exercisable in different years.
The company wants to provide the NQSOs to assist the individual in purchasing the ISOs.
Since they are virtually the same there is no compelling reason to issue both in the same year.

A

The company wants to provide the NQSOs to assist the individual in purchasing the ISOs.
- When both ISOs and NQSOs are available in the same year the individual can exercise and sell the unfavored NQSOs to generate enough cash to purchase and hold the favored ISOs. It would also be valuable to have both if they issued over $100,000 in options exercisable in the same year because there is a $100,000 limit on ISOs.

How well did you know this?
1
Not at all
2
3
4
5
Perfectly
39
Q

David is awarded an immediately vested, non-qualified stock option for 1,000 shares of company stock with an exercise price of $35 per share while the stock price is currently $33 per share. What are the tax ramifications, if any at the date of the grant?

$0
The gain between exercise and actual price of $2,000 is immediately taxable.
The awarded option price value of $33,000 will be immediately taxable.
Because these are unrealized gains, neither the option value nor the gain are taxable until the stock is finally sold.

A

$0
- In the case of NQS Options, the option is not taxed at the grant if the exercise price is equal to or greater than the fair market value of the stock.

How well did you know this?
1
Not at all
2
3
4
5
Perfectly
40
Q

Which of the following statements concerning cash balance pension plans is correct?

The cash balance plan is a defined benefit plan because the annual contribution is defined by the plan as a percentage of employee compensation.
The cash balance plan provides a guaranteed annual investment return to participant’s account balances that can be fixed or variable and is 100% guaranteed by the Pension Benefit Guarantee Corporation.
The cash balance plan uses the same vesting schedules as traditional defined benefit plans.
The adoption of a cash balance plan is generally motivated by two factors: selecting a benefit design that employees can more easily understand than a traditional defined benefit plan, and as a plan that has more predictable costs associated with its funding.

A

The adoption of a cash balance plan is generally motivated by two factors: selecting a benefit design that employees can more easily understand than a traditional defined benefit plan, and as a plan that has more predictable costs associated with its funding.
- Cash balance plans are defined benefit plans due to the guaranteed investment returns and benefit formula, not simply a contribution amount. While cash balance plans provide guaranteed rates of return, they are not 100% guaranteed by the PBGC (PBGC has coverage limits). Cash balance plans use 3-year cliff vesting only. Choice d is correct.

How well did you know this?
1
Not at all
2
3
4
5
Perfectly
41
Q

Abe’s Apples has an integrated defined benefit pension plan. The plan currently funds the plan using a funding formula of Years of Service × Average of Three Highest Years of Compensation × 1.5%. If Geoffrey has been there for 40 years what is the maximum disparity allowed using the excess method?

.75%
5.7%
26.25%
60%

A
  1. 25%
    - The maximum disparity using the excess method is the lesser of the formula amount (40 years × 1.5%) or 26.25% (35 years × .75%). 35 years and .75% are the maximums that can be used under the excess method. Note: This level of knowledge is probably not tested on a regular basis, however, because it is part of the board’s topic list this question was added to ensure that you could answer it if it came up on the test.
How well did you know this?
1
Not at all
2
3
4
5
Perfectly
42
Q

WestN, Inc. sponsors a 401(k) profit sharing plan with a 50% match. In the current year, the company contributed 20% of each employee’s compensation to the profit sharing plan in addition to the match to the 401(k) plan. The company also allocated a forfeiture allocation of $4,000. The ADP of the 401(k) plan for the NHC is 4%. Wade, who is age 45, earns $190,000 and owns 19% of the company stock. If Wade wants to maximize the contributions to the plan, how much will he defer into the 401(k) plan?

$19,000
$11,400
$9,333
$4,667

A

$9,333
- Wade is highly compensated because he is more than a 5% owner, so the maximum that he can defer to satisfy the ADP Test requirements is 6% (4% + 2%). Wade is also limited by the 415(c) limit of $56,000. Since the company contributes $42,000 (20% of $190,000 + $4,000 of forfeiture allocations), he only has $14,000 to split between the deferral and the match. Thus, he contributes $9,333* and the match is $4,667, which when added to the $42,000 totals $56,000. 6% of his salary of $190,000 is $11,400. However, he cannot defer this amount due to the 415(c) limit.

How well did you know this?
1
Not at all
2
3
4
5
Perfectly
43
Q

Spenser is covered under his employer’s top-heavy New Comparability Plan. The plan classifies employees into one of three categories: 1) Owners, 2) Full-time employees, 3) Part-time employees. Assume the IRS has approved the plan and does not consider it to be discriminatory. The employer made a 4% contribution on behalf of all owners, 2% contribution on behalf of all Full-time employees and 1% contribution on behalf of all part time employees. If Spenser currently earns $50,000 per year and is a full-time employee, what is the contribution that should be made for him?

$1,000
$1,500
$18,000
$54,000

A

$1,500
- For a profit sharing plan the contribution is limited to the lesser of $56,000 (2019) or covered compensation. Since the plan is top heavy, the plan must provide a benefit to all non-key employees of at least 3%, therefore; 50,000 × 3% = $1,500.

How well did you know this?
1
Not at all
2
3
4
5
Perfectly
44
Q

Which of the following is not a qualified retirement plan?

ESOP.
401(k) plan.
403(b) plan.
Target benefit plan.

A

403(b) plan.
- A 403(b) plan is a tax-advantaged plan (tax qualified), not a qualified retirement plan. All of the others are qualified plans subject to ERISA rules.

How well did you know this?
1
Not at all
2
3
4
5
Perfectly
45
Q

In determining the allowable annual additions per participant to a defined-contribution pension plan account in the current year, the employer may NOT include:

Compensation exceeding $56,000 (indexed).
Compensation exceeding $280,000.
Compensation exceeding the defined-benefit limitation in effect for that year.
Bonuses.

A

Compensation exceeding $280,000.
- Option “A” - $56,000 is the maximum contribution. Option “C” - Defined contribution plans do not have defined benefits. Option “D” - Bonuses are includible as compensation if the plan so provides.

How well did you know this?
1
Not at all
2
3
4
5
Perfectly
46
Q

Cody is considering establishing a 401(k) for his company. He runs a successful video recording and editing company that employs both younger and older employees. He was told that he should set up a safe harbor type plan, but has read on the Internet that there is the safe harbor 401(k) plan and a 401(k) plan with a qualified automatic contribution arrangement. Which of the following statements accurately describes the similarities or differences between these types of plans?

The safe harbor 401(k) plan has more liberal (better for employees) vesting for employer matching contributions as compared to 401(k) plans with a qualified automatic contribution arrangement.
Both plans provide the same match percentage and the same non-elective contribution percentage.
Employees are required to participate in a 401(k) plan with a qualified automatic contribution arrangement.
Both types of plans eliminate the need for qualified matching contributions, but may require corrective distributions.

A

The safe harbor 401(k) plan has more liberal (better for employees) vesting for employer matching contributions as compared to 401(k) plans with a qualified automatic contribution arrangement.
- Safe harbor plans require 100% vesting, while 401(k) plans with QACAs require two year 100% vesting. The matching contributions are different for the plans. Employees are not required to participate in either plan. Both plans eliminate the need for ADP testing, which means that they eliminate the need for qualified matching contributions and corrective distributions.

How well did you know this?
1
Not at all
2
3
4
5
Perfectly
47
Q

Timothy is covered under his employer’s Defined Benefit Pension Plan. He earns $500,000 per year. The Defined Benefit Plan uses a funding formula of Years of Service × Average of Three Highest Years of Compensation × 2%. He has been with the employer for 25 years. What is the maximum contribution that can be made to the plan on his behalf?

$132,500
$225,000
$280,000
It is indeterminable from the information given.

A

It is indeterminable from the information given.
- Read the question carefully. The question asks “what is the maximum contribution that can be made.” Remember that for a defined pension plan the contribution must be whatever the actuary determines needs to be made to the plan.

How well did you know this?
1
Not at all
2
3
4
5
Perfectly
48
Q

Which of the following statements accurately reflects the overall limits and deductions for employer contributions to qualified plans?

I. An employer’s deduction for contributions to a money purchase pension plan and profit sharing plan is limited to the lesser of 25% of covered payroll or the maximum Section 415 limits permitted for individual account plans.
II. An employer’s deduction for contributions to a defined benefit pension plan and profit sharing plan cannot exceed the lesser of the amount necessary to satisfy the minimum funding standards or 25% of covered payroll.
III. Profit sharing minimum funding standard is the lesser of 25% or the Section 415 limits permitted for individual account plans.

I only.
I and II only.
II and III only.
I, II and III.

A

I only.
- Statement “II” is incorrect because there is no 25% of covered payroll limitation in a DB plan. Statement “III” is incorrect because there is no minimum funding standard for profit sharing plans.

How well did you know this?
1
Not at all
2
3
4
5
Perfectly
49
Q

Lisa, age 35, earns $175,000 per year. Her employer, Reviews Are Us, sponsors a qualified profit sharing 401(k) plan, which is not a Safe Harbor Plan, and allocates all plan forfeitures to remaining participants. If in the current year, Reviews Are Us makes a 20% contribution to all employees and allocates $5,000 of forfeitures to Lisa’s profit sharing plan account, what is the maximum Lisa can defer to the 401(k) plan in 2019 if the ADP of the non-highly employees is 2%?

$7,000
$13,000
$16,000
$19,000

A

$7,000
- The maximum annual addition to qualified plan accounts is $56,000. If Reviews Are Us contributes $35,000 ($175,000 × 20%) to the profit sharing plan and Lisa receives $5,000 of forfeitures, she may only defer $16,000 ($56,000 - $35,000 - $5,000) before reaching the $56,000 limit (2019). However, she will also be limited by the ADP of the non-highly employees because she is highly compensated (compensation greater than $125,000). If the non-highly employees are deferring 2% then the highly compensated employees can defer 4% (2×2=4). Therefore, she is limited to a deferral of $7,000 ($175,000 × 4%).

How well did you know this?
1
Not at all
2
3
4
5
Perfectly
50
Q

How do cash balance plans differ from traditional defined benefit pension plans?

Traditional defined benefit plans are required to offer payment of an employee’s benefit in the form of a series of payments for life while cash balance plans are not.
Traditional defined benefit plans define an employee’s benefit as a series of monthly payments for life to begin at retirement, but the cash balance plan defines the benefit in terms of a stated account balance.
In Cash Benefit Plans, these accounts are often referred to as hypothetical accounts because they do not reflect actual contributions to an account or actual gains and losses allocable to the account, whereas in a Defined Benefit Pension Plan they do.
Pension Plans are available to retirees in a lump sum payment, whereas Cash Balance Plans are not.

A

Traditional defined benefit plans define an employee’s benefit as a series of monthly payments for life to begin at retirement, but the cash balance plan defines the benefit in terms of a stated account balance.
- Answer “A” is incorrect because Cash Benefit Plans are required to offer payment of an employee’s benefit in the form of a series of payments for life. Answer “C” is incorrect, because neither plan shows actual gains or losses allocable to the account. Answer “D” is incorrect and stated exactly opposite of how it is in fact.

How well did you know this?
1
Not at all
2
3
4
5
Perfectly
51
Q

Carol, age 55, earns $200,000 per year. Her employer, Reviews Are Us, sponsors a qualified profit sharing 401(k) plan, which is not a Safe Harbor Plan, and allocates all plan forfeitures to remaining participants. If in the current year, Reviews Are Us makes a 18% contribution to all employees and allocates $7,000 of forfeitures to Carol’s profit sharing plan account, what is the maximum Carol can defer to the 401(k) plan in 2019 if the ADP of the non-highly employees is 1%?

$4,000
$19,000
$10,000
$25,000

A

$10,000
- The maximum annual addition to qualified plan accounts is $56,000. If Reviews Are Us contributes $36,000 ($200,000 × 18%) to the profit sharing plan and Lisa receives $7,000 of forfeitures, she may only defer $13,000 ($56,000 - $36,000 - $7,000) before reaching the $56,000 limit. However, she will also be limited by the ADP of the non-highly employees because she is highly compensated (compensation greater than $125,000). If the non-highly employees are deferring 1% then the highly compensated employees can defer 2% (1×2=2). Therefore, she is limited to a deferral of $4,000 (200,000 x 2%). Since she is 50 or older she can also defer the catch-up amount of $6,000 which is not subject to the ADP limitation. Therefore, her maximum deferral is $10,000.

How well did you know this?
1
Not at all
2
3
4
5
Perfectly
52
Q

Matt is a participant in a profit sharing plan which is integrated with Social Security. The base benefit percentage is 6%. Which of the following statements is/are true?

I, The maximum permitted disparity is 100% of the base benefit level or 5.7%, whichever is lower.
II. The excess benefit percentage can range between 0% and 11.7%.
III. Elective deferrals may be increased in excess of the base income amount.
IV. The plan is considered discriminatory because it gives greater contributions to the HCEs.
I and II only.
I, II and IV only.
II only.
I, II, III and IV.

A

I and II only.
- His base rate is 6% and the social security maximum disparity is 5.7% for 11.7% as the top of his range.

Statement “III” is incorrect because integration does not affect voluntary deferrals by employees. Statement “IV” is incorrect because, done properly, integration is NOT considered discriminatory.

How well did you know this?
1
Not at all
2
3
4
5
Perfectly
53
Q

Which of the following apply to legal requirements for a qualified thrift/savings plan?

Participants must be allowed to direct the investments of their account balances.
Employer contributions are deductible when contributed.
In-service withdrawals are subject to financial need restrictions.
After-tax employee contributions cannot exceed the lesser of 100% of compensation or $56,000.

A

After-tax employee contributions cannot exceed the lesser of 100% of compensation or $56,000.
- This correctly describes the Section 415(c) limits on maximum contributions permitted by law. Participants do not have to be given the right to direct their investments. Employees make after-tax contributions to a thrift; employers don’t make contributions. Answer “C” is incorrect because only 401(k) plans have statutory hardship withdrawal requirements, not thrift/savings plans.

How well did you know this?
1
Not at all
2
3
4
5
Perfectly
54
Q

Generally, which of the following are noncontributory plans?

I. 401(k) and money purchase pension plans .
II. 401(k) and thrift plans.
III. Thrift plans and ESOPs.
IV. Money purchase pension plans and profit sharing plans.
IV only.
I and II only.
III and IV only.
I, II, III and IV.
A

IV only.
- Employers generally contribute to Money Purchase Pension Plans, ESOPs, and Profit Sharing Plans. Employees contribute (thus contributory plans) to 401(k)s and Thrift Plans.

How well did you know this?
1
Not at all
2
3
4
5
Perfectly
55
Q

Which of the following statements concerning stock bonus plans and ESOPs is(are) true?

I. They both give employees a stake in the company through stock ownership and allow taxes to be delayed on stock appreciation gains.
II. They both limit availability of retirement funds to employees if an employer’s stock falls drastically in value and create an administrative and cash-flow problem for employers by requiring them to offer a repurchase option (a.k.a. put option) if their stock is not readily tradable on an established market.
I only.
II only.
I and II.
Neither I or II.

A

I and II.

- Statement “I” lists advantages of choosing stock ownership plans and ESOPs. Statement “II” lists the disadvantages.

How well did you know this?
1
Not at all
2
3
4
5
Perfectly
56
Q

Which of the following vesting schedules may a non-top-heavy profit sharing plan use?

I. 2 to 6 year graduated.
II. 3-year cliff.
III. 1 to 4 year graduated.
IV. 3 to 7 year cliff.

I only
II and III only
I, II and III only
I, II, III and IV

A

I, II and III only
- As a result of the PPA 2006, a profit sharing plan must vest at least as rapidly as a 3-year cliff or 2 to 6 year graduated schedule without regard to the plan’s top-heavy status. The profit sharing plan can follow any vesting schedule that provides a more generous vesting schedule.

How well did you know this?
1
Not at all
2
3
4
5
Perfectly
57
Q

Select those statements which accurately reflect characteristics of defined contribution pension plans?

I. Allocation formula which is indefinite.
II. Account value based benefits.
III. Employer contributions from business earnings.
IV. Fixed employer contributions based upon terms of plan.

I and II only.
II and III only.
II and IV only.
I, II and III only.

A

II and IV only.
- Defined contribution pension plans must have a definite allocation formula based upon salary and/or age or any other qualifying factor. Contributions may be made without regard to company profits and, because it is a pension plan, are fixed by the funding formula and must be made annually.

How well did you know this?
1
Not at all
2
3
4
5
Perfectly
58
Q

J.P. is covered under his employer’s Profit Sharing Plan. He currently earns $500,000 per year. The plan is top heavy. The employer made a 10% contribution on behalf of all employees. What is the maximum retirement benefit that can be paid to him?

$24,000
$50,000
$54,000
Cannot be determined by the information given.

A

Cannot be determined by the information given.
- Read the question carefully. The question asks “what is the maximum retirement benefit that can be paid to him.” Remember that for a profit sharing plan the contribution to an employer is limited to the lesser of $56,000 (2019) or covered compensation however, the actual retirement benefit will be whatever is in the account balance at the time of retirement.

How well did you know this?
1
Not at all
2
3
4
5
Perfectly
59
Q

Which one of the following statements is NOT correct?

Profit-sharing plans fall under the broad category of defined contribution plans.
Profit-sharing plans are best suited for companies which have unstable cash flows.
A company which adopts a profit-sharing plan is required to make annual contributions to the plan.
The maximum tax-deductible employer contribution to a profit-sharing plan is 25% of covered compensation.

A

A company which adopts a profit-sharing plan is required to make annual contributions to the plan.
- The employer is not required to make any particular percentage of profits. Though contributions must be substantial and recurring, the plan concerns itself more with allocation requirements rather than with contributions. As long as contributions are recurring, they need not be made in a year where the employer has not made a profit.

How well did you know this?
1
Not at all
2
3
4
5
Perfectly
60
Q

Shane’s Rib Shack has a Target Benefit Plan. They have 10 employees with the following compensations: Employee Compensation

$300,000
$100,000
$75,000
$50,000
$50,000
$50,000
$50,000
$25,000
$25,000
$20,000
Based on the actuarial table that was established at the inception of the plan they should fund the plan with $210,000. What is the maximum deductible contribution that can be made to the plan?

$181,250
$186,250
$195,000
$210,000

A

$181,250
- Since the plan is a defined contribution plan the maximum deductible contribution is 25% of the total covered compensation. The max covered compensation of all employees is $725,000. Thus the maximum deductible limit is $181,250 ($725,000 × 25%). Remember to limit employee 1 to the $280,000 (2019) covered compensation limit. The actuarial table amount is irrelevant because this a defined contribution plan.

How well did you know this?
1
Not at all
2
3
4
5
Perfectly
61
Q

Robin just started at Financial University Network (FUN) and has been encouraged by several of the “old timers” to save part of her salary into the 401(k) plan. She is not yet convinced as she likes to shop. Which of the following statements is accurate regarding 401(k) plans?

A 401(k) plan must allow participants to direct their investments.
Deferrals into the 401(k) plan must be contributed by the end of the following calendar quarter into the plan.
Employees that join the plan must be provided with a summary plan description.
A 401(k) plan is financially safe because it must have an annual audit.
A

Employees that join the plan must be provided with a summary plan description.
- Answer c is correct as employees must be given a summary plan description, which provides basic information about the operation of the plan. Answer a is not correct as some 401(k) plans may have the asset managed by an investment manager. However, most 401(k) plans will provide for employee self directing of their 401(k) balances. Answer d is not correct as there is no requirement for an annual audit of a 401(k) plan.

How well did you know this?
1
Not at all
2
3
4
5
Perfectly
62
Q

An actuary establishes the required funding for a defined benefit pension plan by determining:

The lump sum equivalent of the normal retirement life annuity benefit of each participant.
The amount of annual contributions needed to fund single life annuities for the participants at retirement.
The future value of annual employer contributions until the participant’s normal retirement date, taking an assumed interest rate, the number of compounding periods, and employee attrition into account.
The amount needed for the investment pool to fund period certain annuities for each participant upon retirement.

A

The amount of annual contributions needed to fund single life annuities for the participants at retirement.
- Statement “A” is incorrect because it deals with a lump sum, NOT annual contributions. Statement “C” is incorrect because DB plans deal with present value calculations, not future values. Statement “D” is incorrect because DB plans deal with life annuities, NOT period certain annuities.

How well did you know this?
1
Not at all
2
3
4
5
Perfectly
63
Q

One of your clients wants to know the maximum amount that might be allocated to her 401(k) account in the current year. She expects to earn $80,000. What amount can you tell your client will be the maximum total annual additions which could be made to her account?

$19,000, the employee elective deferral
$20,000, 25% of income
$25,000
$56,000

A

$56,000
- The section 415 limit is applied to all annual additions. The lesser of 100% of income or $56,000 (2019) is the restriction. This amount includes all company contributions and salary deferral maximums.

How well did you know this?
1
Not at all
2
3
4
5
Perfectly
64
Q

Calculate the maximum contribution (both employer and employee elective deferrals) for an employee (age 39) earning $285,000 annually, working in a company with the following retirement plans: a 401(k) with no employer match and a money-purchase pension plan with an employer contribution equal to 12% of salary.

$19,000
$33,600
$52,600
$56,000

A

$52,600
- For the purposes of this calculation, the compensation exceeding $280,000 is not recognized. The employer is contributing 12% of $280,000 (or $33,600) for the money purchase plan and the employee may contribute up to $19,000 in 2019 to the 401(k) plan. This totals $52,600.

How well did you know this?
1
Not at all
2
3
4
5
Perfectly
65
Q

Match the following statement with the type of retirement plan which it most completely describes: “A plan which requires annual employer contributions equal to a formula determined by each participant’s salary” is a…

Profit sharing plan.
Money purchase plan.
SIMPLE IRA.
Defined benefit plan.

A

Money purchase plan.
- Defined benefit plan and cash balance plan (Answer “D”) contributions are determined by age, as well as salary. Answer “A” doesn’t require annual contributions. Answer “C” has employer contributions determined by the amount of employee deferrals.

How well did you know this?
1
Not at all
2
3
4
5
Perfectly
66
Q

Target benefit plans and defined benefit plans have which of the following characteristics in common?

Minimum funding standards apply.
Qualified joint and survivor annuity requirements apply.
High investment earnings increase participant retirement benefits.
The employer is obligated to provide a specified benefit in retirement.
I and II only.
I and III only.
II and IV only.
III and IV only.

A

I and II only.
- Both plans are pension plans, therefore Statements “I” and “II” apply. Statement “III” applies only to the target benefit plan (because it is a DC plan) NOT the DB plan. Statement “IV” applies only to the DB plan NOT the target benefit plan.

67
Q

Dr. Dylan James is a 55-year-old Doctor who just started his gastroenterologist practice and hired Nurse Nancy, who is age 25. Dr. DJ is expected to earn annual income of $350,000 that will increase at least at the rate of inflation. Inflation is expected to be 3 percent. Which of the following defined benefit plan formulas would you recommend if Dr. DJ wants to maximize his benefits in retirement, which is expected to occur at age 65?

Unit benefit (a.k.a. percentage-of-earnings-per-year-of-service) formula.
Flat-percentage formula.
Flat-amount formula.
New comparability formula.

A

Flat-percentage formula.
- The unit credit formula rewards many years of service. The flat percentage formula will work well, as long as the Doctor has ten years of service. The maximum benefits under IRC 415(b) are reduced for participation less than 10 years. The flat amount would provide higher benefits for Nurse Nancy compared to Dr. DJ on comparative basis. A new comparability plan is a profit sharing plan.

68
Q

Sherman, age 52, works as an employee for Cupcakes Etc, a local bakery. Cupcakes sponsors a 401(k) plan. Sherman earns $50,000 and makes a 10% deferral into his 401(k) plan. His employer matches the first 3% deferral at 100% and they also made a 5% profit sharing contribution to his plan. Sherman also owns his own landscaping business and has adopted a solo 401(k) plan. His landscaping business earned $40,000 for the current year. What is the total contribution that can be made to the solo plan, assuming his self-employment taxes are $6,000?

$19,000
$20,000
$25,000
$27,400

A

$27,400
An individual can defer up to $19,000 (2019) plus an additional $6,000 catch up for all of their 401(k) and 403(b) plans combined. Since he is 50 or older he can contribute the 19,000 + 6,000 = $25,000. Since he already contributed $5,000 into his employer plan he can still defer $20,000 ($25,000 - $5,000) into the solo plan. The employer contributions in this question are in addition to the employee deferral limit.

Employer contribution into the solo plan:

self-employment income $40,000
less 1/2 SE tax $3,000
Net $37,000
X 20%
employer contribution $7,400
Total contribution to the solo plan = $20,000 + $7,400
69
Q

Which of the following is not true of a short-term disability plan?

Has a more generous definition of disability than long-term plans.
Premiums paid under an insured plan are deductible by the employer.
Payments made by an employee are generally not tax deductible.
Benefit payments under an employer-paid plan are tax exempt to the employee.

A

Benefit payments under an employer-paid plan are tax exempt to the employee.
- Premiums under an employer-paid plan are deductible to the employer when paid to the insurance company. The employee must claim the benefits from the employer-paid policy as taxable income. If employees pay for the premium on an after-tax basis, benefits are tax exempt. Premiums paid by the employee are deductible only though a Section 125 cafeteria plan, then benefits are taxable. Definitions of disability are much more liberal under short-term disability than under long-term disability.

70
Q

Funds in an HSA can be used for which of the following:

I. Qualified medical expenses
II. Deductibles
III. Vision care
IV. Dental expenses

I and II only.
II and III only.
I, III and IV only.
I, II, III and IV.

A

I, II, III and IV.

- All of these expenses are eligible under health savings plans.

71
Q

Which of the following correctly describes characteristics of group universal life insurance?

I. The contract has a master group policy.
II. The employer usually pays all of the policy premiums.
III. Expenses are often lower than for individual universal life policies.
IV. These policies offer the potential for higher returns than whole life policies.
V. The coverage is based on a combination of decreasing units of group term and accumulating units of single premium whole life.
I and II only.
III and IV only.
IV and V only.
I, II and III only.

A

III and IV only.
- Statement “I” applies to group term life. Statement “II” is false. Usually the employee is required to pay part or all of the premium cost of group universal life insurance. Statement “V” applies to a group whole life program.

72
Q

Dues to business-related organizations provided as a fringe benefit are:

Includable in taxable income of all covered employees.
Includable in the taxable income of key employees only.
Excludable from the taxable income of all covered employees.
Excludable from the taxable income of non-highly compensated employees only.

A

Excludable from the taxable income of all covered employees.

- Dues and licenses are excluded from taxable income if directly related to the employee’s job.

73
Q

Which of the following is/are accurate of a Section 125 cafeteria plan?

I. 30% of the total benefits can accrue to key employees.
II. There must be at least one cash benefit.
III. Deferral of income is not allowed except through a 403(b).
IV. Salary reductions can be changed at any time during the year.
I only.
II only.
II and III only.
I and IV only.

A

II only.
- Statement “I” is incorrect because only 25% of the total benefits can accrue to key employees. Statement “III” is incorrect because deferrals are allowed only through a 401(k) plan. Statement “IV” is incorrect because mid-year changes in reductions are allowed only for qualified changes in status.

74
Q

Many employers are now making flexible spending accounts (FSAs) available to employees. Which of the following statements concerning the nature of these accounts is incorrect?

The balance in an employee’s FSA can be carried forward or exchanged for cash if unused for eligible expenses incurred during the plan year.
An FSA is technically a cafeteria plan benefit that can be used by itself or as part of a broader cafeteria plan.
A separate FSA salary reduction must be made for each type of eligible benefit.
A salary reduction for an FSA will lower an employee’s income for social security tax purposes if the employee earns less than the social security wage base.

A

The balance in an employee’s FSA can be carried forward or exchanged for cash if unused for eligible expenses incurred during the plan year.
- Cafeteria plans have a “use-it-or-lose-it” provision which requires any funds not used to pay qualified claims during the plan year be forfeited back to the plan sponsor. Forfeited funds cannot be rebated back to the individual employee who forfeited the funds. Answer “B” is correct because a cafeteria plan must have only two benefits, one taxable (salary) and one pre-tax (FSA). Answers “C” and “D” are also true.

75
Q

Which of the following statements concerning COBRA is correct?

An employer’s plan is exempt from COBRA provisions if the employer averages 25 or fewer employees.
Continuation coverage must be available to terminating employees, but not to full-time employees shifting to part-time status.
After 36 months, the maximum period for continuation of coverage terminates.
The government imposes a non-compliance fine on the employer equal to $10 per day per participant.

A

After 36 months, the maximum period for continuation of coverage terminates.
- Employers must provide COBRA if they have 20 or more employees. A change in benefit status will trigger COBRA eligibility. COBRA non-compliance carries a penalty of $100 per day per participant.

76
Q

Jane P. Lane is a clerical worker who has been with her employer for the last 20 years. Last year, she got married in the Swiss Alps, which was quite out of character for her.

She participates in an employer-paid group term life plan and selected term insurance in the amount of $200,000, which is three times her salary.

She has named her spouse as the beneficiary of the policy.

What is the tax consequence of this policy?

Her employer is permitted to deduct the premiums paid on the entire amount of coverage.
Her employer is permitted to deduct the premiums paid on the first $50,000 of coverage.
Jane is subject to tax on the entire benefit.
Jane is subject to tax on the amount which exceeds three times her annual salary or $50,000, whichever is less.

A

Her employer is permitted to deduct the premiums paid on the entire amount of coverage.
- The employer is permitted to deduct 100% of the premiums, but Jane is subject to taxation on the amount in excess of $50,000.

77
Q

James and Cheryl Hansen, both age 42, are married with 9-year-old triplets. James is an attorney and makes $65,000 per year; he is not a partner. Cheryl earns an annual salary of $15,000 as a teacher’s aide at the private, for-profit school the triplets attend. James’ firm provides group permanent whole life and group survivor income benefit insurance. The school provides Cheryl with group term insurance coverage. James’ Group Permanent Whole Life - This is a non-discriminatory plan designed to provide supplemental income to James during retirement. The law firm pays the entire premium for the plan. Cheryl is the designated beneficiary on the policy. The policy indicates that James has full vesting in the ownership of the policy. James’ Group Survivor Income Benefit Insurance - This is a payroll deduction plan with James paying the entire premium. The plan is not a discriminatory plan. James has elected the survivor benefit that will pay benefits to his spouse and to any children under the age of 21. If James were to die today, the following benefits would be paid annually to the beneficiaries: If spouse is sole beneficiary - 30% of salary. If children are sole beneficiaries - 40% of salary. If spouse and children are beneficiaries - 50% of salary. Cheryl’s Basic Life Insurance Plan - This is a non-discriminatory, non-contributory plan. The death benefit equals five times annual salary for employees age 55 or younger, three times the salary for those employees over age 55. James is the designated beneficiary on the policy. Given the above information, which of the following statements are correct about James and Cheryl’s group insurance coverage?

I. At James’ death, the group survivor income benefit would total $26,000 per year.
II. Assuming James dies, the group survivor income benefits received by his beneficiaries would be taxed as an annuity.
III. Cheryl would be taxed on a portion of the cost of the $75,000 of group term life insurance coverage over $50,000.
IV. At Cheryl’s death, James would be required to include in gross income the proceeds from the group term policy.

I and II only.
II and III only.
II and IV only.
II, III and IV only.

A

II and III only.
- Statement “I” is incorrect. The benefit would actually be $32,500 ($65,000 × 50%). Statement “IV” is incorrect because the death benefits are generally free from income tax. (For exceptions review the transfer for value rules.) Statements “II” and “III” are correct. The survivor income would be allocated between cost basis (cumulative premiums paid on an after-tax basis divided by the projected number of payments).

78
Q

Evaluate the following statements:

I. De minimis fringe benefits are those that are so immaterial that . . .
II. De minimis fringe benefits are subject to strict anti-discrimination . . .

I and II are true.
I is true, but II is false.
I and II are false.
I is false, but II is true.

A

I is true, but II is false.
- The de minimis exemption is not subject to a non-discrimination requirement because the amounts are too small to make it worthwhile to account for the items.

79
Q

Which of the following is a correct statement about the income tax implications of employer premium payments for group health insurance?

An S Corporation can only deduct 70% of the premiums for all employees.
In a sole proprietorship, the premiums for both the owner and the non-owner are fully deductible.
If stockholder/employees of a closely held C corporation are covered as employees, the premiums are fully deductible.
Premium costs paid by a partnership are passed through to the partner, who can deduct 70% of the costs on their individual tax returns.

A

If stockholder/employees of a closely held C corporation are covered as employees, the premiums are fully deductible.
- S Corporations and proprietorships cannot deduct any premiums for group health insurance for owners. Non-owner employee health premiums are fully deductible to both entities. Answer “D” is incorrect because partners are able to deduct 100% of the health insurance premium on their individual tax returns.

80
Q

Which of the following correctly describes the tax implications of a self-funded accident or medical plan where the employer reimburses the employee directly?

I. In a discriminatory plan, the employer cannot deduct the reimbursements paid to the employee.
II. In a discriminatory plan, a highly-compensated employee must include the excess benefit in his or her income.
III. In a non-discriminatory plan, the benefits received by employees are generally tax free without limitation.
IV. In a non-discriminatory plan, the employer can deduct reimbursements to the employee if they are paid to the employee or the employee’s beneficiary and are considered reasonable compensation.
V. In a discriminatory plan, benefits received by non-highly compensated employees are generally tax free without limit.
I, II and IV only.
II, III and IV only.
III, IV and V only.
I, III, IV and V only.

A

III, IV and V only.
- Statement “I” is incorrect because the employer can always deduct the premiums. Statement “II” - The highly compensated employees may be required to pay taxes on all or part of the reimbursements.

81
Q

Cafeteria plans have which of the following characteristics?

I. Must offer a choice between at least one qualified “pre-tax” benefit and one non-qualified “cash” benefit.
II. Medical Flexible Spending Accounts (FSAs) can reimburse medical expenses not covered by insurance for the participant and all dependents.
III. Changes in election amount during the plan year can only occur with a “qualifying change in family status.”
IV. Salary reductions are not subject to income taxes but payroll taxes apply.

I, II and IV only.
II, III and IV only.
I, II and III only.
I, II, III and IV.

A

I, II and III only.
- Cafeteria Plans (Section 125) allow salary reductions which are taken from an employee’s salary before Federal and State withholding tax as well as Social Security and Medicare taxes (FICA). At least one taxable and non-taxable benefit must be offered under a plan. Medical FSAs allow reimbursement for eligible medical expenses for the employee and any dependents. A qualifying change in status is required to make a mid-year change in elections.

82
Q

The Health Insurance Portability and Accountability Act of 1996 (HIPAA) impacts an employee and employer in which of the following ways:

I. An employee without creditable coverage can generally only be excluded by the group health insurance plan (if offered) for up to twelve months.
II. The waiting period is reduced by the amount of “creditable coverage” at a previous employer.
III. If the employee does not enroll in the group health insurance plan at the first opportunity, an 18-month exclusion period may apply.

I and II only.
I, II and III only.
II and III only.
II only.

A

I, II and III only.
- All three statements are true. If you have a pre-existing condition that can be excluded from your plan coverage, then there is a limit to the pre-existing condition exclusion period that can be applied. HIPAA limits the pre-existing condition exclusion period for most people to 12 months (18 months if you enroll late), although some plans may have a shorter time period or none at all. In addition, some people with a history of prior health coverage will be able to reduce the exclusion period even further using “creditable coverage.” People with a history of prior health coverage will be able to reduce the exclusion period even further using “creditable coverage.”

83
Q

Which of the following accurately describes some attributes of non-qualified retirement plans?

I. The employee will pay Table 1 costs each year on an “employer pay all” split dollar life insurance arrangement.
II. The employer can deduct the premiums paid for a split-dollar life insurance arrangement in the year the premiums are paid.
III. Death benefits from a split-dollar arrangement, both the employer and the employee’s beneficiary’s share, are generally tax free.
IV. If the employee’s portion of the life insurance premium is greater than the P.S. 58 cost, the excess premiums “rolls forward” to a future year to accurately reflect the employee’s cost basis.
I and III only.
II and IV only.
I and IV only.
I and II only.

A

I and III only.
- Statement “II” is incorrect because the employer is unable to deduct any contributions to a non-qualified plan until the employee actually takes constructive receipt. In the traditional split-dollar arrangement, the employer has an interest in the cash values of the split-dollar policy equal to the amount of premiums paid, and therefore, there is never a deduction for premiums paid. Statement “III” - Because no tax deductions are taken for any premiums paid on the policy, the death benefits are tax-free. Statement “IV” - The employee is required to pay the Table 1 cost each year, without regard to premiums paid in previous years.

84
Q

A non-qualified deferred compensation plan providing the key employee with a vested beneficial interest in an account is known as:

A Supplemental Executive Retirement Plan (SERP).
A funded deferred compensation plan.
An excess benefit plan.
A Rabbi trust.

A

A funded deferred compensation plan.
- If the employee has a non-forfeitable beneficial interest in a deferred compensation account, the IRS considers the plan “funded” and subject to current income tax due because the employee has constructive receipt of the assets.

85
Q

Which of the following statements concerning tax considerations of nonqualified retirement plans is/are correct?

I. Under IRS regulations an amount becomes currently taxable to an executive even before it is actually received if it has been “constructively received.”
II. Distributions from nonqualified retirement plans are generally subject to payroll taxes.

I only.
II only.
I and II.
Neither I or II.

A

I only.
- Statement “II” is incorrect because payroll taxes are due on deferred compensation at the time the compensation is earned and deferred, not at the date of distribution. Statement “I” is a correct statement.

86
Q

Which of the following accurately describe the results of “golden parachute” payments made to a “disqualified” person?

I. They are includible in W-2 income.
II. They are subject to a 10% excise tax.
III. They qualify for 10-year forward averaging if paid out as a lump sum.
IV. They are not subject to payroll taxes.

I only.
II only.
II and III only.
I and IV only.

A

I only.
- Payments under a “golden parachute” are considered ordinary income. Additionally, any amounts under the Social Security cap will be subject to the OASDI tax. All amounts will be subject to Medicare tax. “Golden parachute” payments are also subject to an additional 20% excise tax. Because these are non-qualified plans, no lump sum treatment or IRA rollover options apply.

87
Q

Which of the following statements concerning the basic characteristics of non-qualified deferred compensation plans is accurate?

The employer immediately deducts contributions made into the informal funding arrangement in any given year.
During their working years, employees have the same tax treatment under a non-qualified plan as under a qualified plan.
Under both an unfunded plan and a formally funded plan, the only security or guarantee the employee has is the unsecured promise of the employer.
Under both the funded plan and an informally funded plan, the IRS may likely rule that the employee is in constructive receipt of income unless there is a substantial risk of loss or forfeiture.

A

Under both the funded plan and an informally funded plan, the IRS may likely rule that the employee is in constructive receipt of income unless there is a substantial risk of loss or forfeiture.
- Employer contributions into a non-qualified plan are not tax deductible to the employer until the employee has constructive receipt and is taxed on the income. Most non-qualified plans are designed to avoid constructive receipt by the employee until retirement. In an informally funded plan (Rabbi trust), the employee has the segregated assets as security of the agreement, assuming the employer remains solvent and the assets are not taken by the employer’s creditors. This risk of having creditors take the assets inside a “Rabbi trust” is what constitutes a substantial risk of loss or forfeiture and keeps the employee from being considered in “constructive receipt” of the formally funded assets.

88
Q

Qualified and non-qualified retirement plans differ in each of the following except:

Rollover provisions.
Permissible discrimination.
Timing of employer deductibility.
Timing of employee taxability.

A

Timing of employee taxability.
- The timing of employee taxation on properly executed non-qualified and qualified plans are the same. Non-qualified plans can be split dollar plans, executive bonus plans, etc.

89
Q

Which of the following statements does not describe the concept of constructive receipt as it applies to employee benefits and qualified and non-qualified retirement plans?

Funded non-qualified retirement plans do not incur constructive receipt if the agreement was executed prior to the performance of services.
Constructive receipt may occur when the funds or benefits are available or accessible to the employee, regardless of whether the funds are actually received.
Unfunded benefit plans generally avoid constructive receipt because there is a substantial risk of forfeiture.
Salary reduction benefit plans generally avoid constructive receipt if the agreement was executed prior to the performance of services.

A

Funded non-qualified retirement plans do not incur constructive receipt if the agreement was executed prior to the performance of services.
- Generally, funded plans result in constructive receipt because the employee has control of the assets and there is no substantial risk of forfeiture. Keep in mind, the question is looking for the false answer.

90
Q

Which of the following explain the tax ramifications of a non-qualified deferred compensation plan?

I. A participant in an unfunded plan will not be currently taxed if the promise of benefits is unsecured and the agreement is executed prior to the first day of service under the agreement.
II. If a funded plan is established by a general partnership and the benefits for general partners are fully vested, then contributions are deductible to the partnership and each partner receives a pro-rata share of the reduced partnership income.
III. A funded plan with a “Rabbi Trust” will not be currently taxable to the participant, even though vested in the benefits, due to the “substantial risk of forfeiture.”
IV. Payments to be made to a participant’s beneficiary are not included in the decedent’s gross estate if the payments are guaranteed.

I only.
I, II and III only.
I and III only.
II, III and IV only.

A

I and III only.
- Statement “II” is incorrect because any funded plan is taxable unless a Rabbi trust is utilized. Statement “IV” in incorrect because the NPV of the guaranteed future income will be included in the gross estate of the deceased participant.

91
Q

Match the following statement with the type of retirement plan which it most completely describes: “This plan can provide for voluntary participant contributions which must be matched by the employer.”

Profit sharing plan with a 401(k) component.
Money purchase plan.
SIMPLE IRA.
Defined benefit plan.

A

SIMPLE IRA.
- Answers “B” and “D” do not permit employee elective deferrals. The profit sharing plan “A” with 401(k) provisions do not require an employer match.The SIMPLE plan has a mandatory match.

92
Q

Match the following statement with the type of retirement plan which it most completely describes: “The plan permits the employer match to deviate below the required percentage in two of the last five years” is a…

Profit sharing plan.
Money purchase plan.
SIMPLE IRA.
Defined benefit plan.

A

SIMPLE IRA.
- The match for a profit sharing plan with 401(k) provisions can vary every year and there is no required percentage. However, a SIMPLE can vary the match in only 2 of 5 years.

93
Q

Pat established his business one year ago. He has hired two assistants. He would like to set up a retirement benefit plan for himself and his two assistants, who want to make voluntary contributions. He is concerned about cash flows for unforeseen business obstacles and future expansion. Of the following types of retirement plans, which would be the most appropriate for Pat’s business:

401(k) plan.
Money purchase pension plan.
Defined benefit plan.
Profit-sharing plan.

A

401(k) plan.
- A 401(k) plan is the only option which allows voluntary employee elective deferral contributions, as desired by the two assistants.

94
Q

Bertha, who is 54 years old, spent most of her career in the corporate world and now provides consulting services and serves as a director for several public companies. Her total self-employment income is $500,000. She is not a participant in any other retirement plan today. She would like to shelter as much of her self-employment earnings as possible by contributing it to a retirement plan. Which plan would you recommend?

Establish a 401(k) plan.
Establish a target benefit plan.
Establish a Deferred Comp program for himself.
Establish a SEP.

A

Establish a 401(k) plan.
- She will be able to defer $56,000 plus the catch up of $6,000 to the 401(k) plan, where she can only contribute $56,000 to the target benefit plan and the SEP. She cannot set up a deferred compensation plan and defer tax.

95
Q

A hybrid plan that uses a discretionary contribution but adjusts for age is a form of a:

Profit sharing plan.
Money purchase plan.
Cash balance plan.
Defined benefit plan.

A

Profit sharing plan.
- Answers “B,” “C” and “D” all require minimum contribution levels. Answer “A” - Profit sharing plan only requires that contributions be “substantial and recurring.” More specifically, an age-based profit sharing plan would be correct.

96
Q

Bobby’s Bar-b-que wants to establish a social security integrated plan using the offset method. Which of the following plans should he establish?

SIMPLE
ESOP
Money Purchase Pension Plan
Defined Benefit Pension Plan

A

Defined Benefit Pension Plan
- He should establish the Defined Benefit Pension Plan. Only defined benefit plans can use the offset method. The Money Purchase Pension plan is a Defined Contribution Plan and must use the excess method. Simple’s and ESOPs cannot be integrated.

97
Q

Brisco, age 51, is the Executive VP of sales at Doggie Daycare (DD). His base salary is $300,000 with a potential bonus of 50%. Brisco is a participant in his employer’s 401(k) plan and always defers the maximum amount. The DD 401(k) plan has the following features and characteristics: Includes a Roth account that is not a safe harbor plan, but has a 50% match up to 4%. The ADP for the NHC is 4.5%. The plan has $3 million in assets that are managed by two asset management firms. DD also sponsors a defined benefit plan that provides a benefit based on years of service and final salary. The DD DB plan provides for 1.5 percent per year of service for the first 20 years and 2 percent for years above 20, up to a max of 35 years. On the weekends, Brisco paints murals. His entity, Wall Works LLC (WW), is a single owned LLC taxed as a disregarded entity. Brisco would like to establish a retirement plan for the income that he earns in WW. He expects to earn $60,000 ever year in WW and wants to know what the best retirement plan is for his business. Which plan would you recommend for him?

SIMPLE
Defined Benefit Plan
401(k) plan
SEP

A

SEP
- Neither a SIMPLE nor a 401(k) plan will work because he is already deferring $18,200 (6.5% (4.5% + 2%) times $280,000) to his 401(k) plan. Therefore, the choices are a 412(e) plan, which is a defined benefit plan funded with insurance products or a SEP, which is extremely easy to set up and one that he can contribute around $11,000 to annually.

98
Q

George, age 35, works for XZY Brothers, Inc., which is installing a new SIMPLE IRA plan in the current year with the maximum match for this year. George makes $30,000 per year and is eligible to participate in the plan. Which of the following is true?

George can have a maximum of $13,900 placed into his account this year.
George may have a maximum of $4,500 placed into his account this year.
George may have a maximum of $13,000 placed in his account this year.
George may have a maximum of $7,500 placed in his account this year.

A

George can have a maximum of $13,900 placed into his account this year.
- George can put in 100% of salary up to $13,000 (2019). XZY will match dollar for dollar up to 3% of salary ($30,000 × .03 = $900). So a total of $13,900 will be placed into the account. All other statements are false.

99
Q

Lien, age 35, recently left his employer, GoGoRoller, a roller blade manufacturer. He left after 18 months because the working conditions were unbearable. GoGoRoller sponsored a SIMPLE IRA. Lien deferred $3,000 into the plan during his time there and the employer contributed $1,500. When he terminated he withdrew the entire account balance of $4,750. Assuming he is in the 15% tax bracket, what is the tax and penalty consequence for this distribution?

$712.50
$775.00
$1,187.50
$1,900.00

A

$1,900.00
- SIMPLE IRAs require a 25% penalty for early withdraws in the first two years if the participant does not meet any of the early withdrawal exceptions. He does not meet any of the exceptions and the distribution is within the first two years. The breakdown of employee deferrals, employer contributions and earnings is irrelevant. Therefore, his tax and penalty consequence is $1,900 = $4,750 × 40%. The 40% is represented by 15% tax plus 25% penalty.

100
Q

Kyle is 56 and would like to retire in 11 years. He would like to live the “high” life and would like to generate 90% of his current income. He currently makes $150,000 and expects $24,000 (in today’s dollars) in Social Security. Kyle is relatively conservative. He expects to make 8% on his investments, that inflation will be 4% and that he will live until 104. How much does Kyle need at retirement?

$3,631,802
$3,343,308
$3,471,896
$3,480,448

A
$3,471,896
Salary = 111,000 (150,000* 90%) - 24,000 = 111,000
N = 11 years to retirement
I = 4% inflation
PV = 111,000 in salary
FV = 170,879.40
BEG PMT = 170,879.40
N = 37 104 - 67
I = 3.8462 Inflation adjusted rate of return = [(1.08/1.04) - 1] × 100
Solve for PV
Answer = 3,471,896.37

Answer “A” is the wrong payment (150,000 -24,000* 90%) = 113,400.

Answer “B” is ordinary annuity (end mode).

Answer “D” uses 4% for interest (beg mode).

101
Q

Farmer Fred wants to retire in 20 years when he turns 64. Fred wants to have enough money to replace 75% of his current income less what he expects to receive from Social Security at the beginning of each year. Fred’s full benefit at age 67 is $25,000 in today’s dollars. Fred is conservative and wants to assume a 7% annual investment rate of return and assumes that inflation will be 3% per year. Based on his family history, Fred expects that he will live 30 years in retirement. Fred just received his brokerage account statement, which he is using to fund his retirement, and it has a balance of $340,596.44. If Fred currently earns $100,000 per year, approximately how much does he need save at the end of each of the next 20 years to fund his retirement?

$6,486
$10,350
$11,215
$12,000

A

$12,000
Step 1: Determine amount to be funded $100,000 income today 75% WRR $75,000 needs ($20,000*) less social security & pension. $55,000 amount to be funded

Step 2: inflate funds to retirement age PV ($55,000) N 20 i 3.00% Pmt 0 FV $99,336.12

Step 3: PV of retirement annuity Pmt $99,336.12 N 30 i 3.8835% FV - PV ($1,809,946.67)

Step 4: Annual funding amount FV $1,809,946.67 N 20 i 7.00% PV ($340,596.44) Pmt ($12,000)

102
Q

When calculating the Wage Replacement Ratio (WRR), what percentage of income is subtracted for a self-employed individual for Social Security and Medicare Taxes?

  1. 65%
  2. 20%
  3. 30%
  4. 3%
A
  1. 30%
    - This is an important point to stress as many clients are self-employed and pay both employer and employee portions of the tax.
103
Q

Of the following expenditures, which is most likely to increase during retirement?

Automobile expenses.
Medical expenses.
Travel expenses.
Mortgage expense.

A

Medical expenses.
- Each of these expenses may increase or decrease during an individual’s retirement, but the most likely expense to increase during retirement is an individual’s medical expenses.

104
Q

Beth works for MG Inc. and was hired right out of school after graduating with a double major in marketing and advertising four years ago. Beth receives a $12,000 distribution from her designated Roth account in her employer’s 401(k) plan as a result of her being disabled. Immediately prior to the distribution, the account consisted of $15,000 of investment in the contract (i.e., designated Roth contributions) and $5,000 of income. What are the tax consequences of this distribution?

She will have $12,000 of income.
She will have $5,000 of income.
She will have $3,000 of income.
She will have no income tax consequences resulting from the distribution.

A

She will have $3,000 of income.
- Non qualified distributions from a Roth account are subject to tax on a prorata basis. Her total account is the 15,000 invested and the 5,000 of income for a balance of $20,000. Since 75% (15,000/20,000) of the value in the account consists of basis and the remaining 25% consists of earnings (5,000/20,000), that same ratio of basis to income will apply to the $12,000 distribution. It is not a qualified distribution because she has not held the account for at least five years.

105
Q

Mayu made a contribution to his Roth IRA on April 15, 2014 for 2013. This was his first contribution to a Roth IRA. Over the years he has made $20,000 in contributions. On May 15, 2019 he withdrew the entire account balance of $45,000 to pay for his daughter’s college education expense. He is 55 years of age. Which of the following statements is true?

He will not include anything in income and will not be subject to the 10% early withdrawal penalty.
He will include $25,000 in income but will not be subject to the 10% early withdrawal penalty.
He will include $25,000 in income and will be subject to the 10% early withdrawal penalty on $25,000.
He will include $45,000 in income and will be subject to the 10% early withdrawal penalty on $45,000.

A

He will include $25,000 in income but will not be subject to the 10% early withdrawal penalty.
- Roth distributions are tax free if they are made after 5 years and because of 1)Death, 2)Disability, 3) 59.5 years of age, or 4)First time home purchase. Although he met the 5 year rule, he did not meet one of the four qualifying reasons. His distribution does not received tax free treatment. The treatment for a non-qualifying distribution allows the distributions to be made from contributions first, then conversions, then earnings. In this case the distinction in distribution order is irrelevant since he withdrew the entire account balance. However, his contribution will be tax free, leaving only the $25,000 in earnings as taxable income. The 10% penalty does not apply to this distribution since he qualifies for the higher education exception to the penalty.

106
Q

Which of the following retirement plans would permit an employee (filing single status) making $100,000 a year to still make the fully deductible contribution to an IRA in the current year?

401(k)
403(b)
SEP
457

A

457
- IRC Section 457 plans are nonqualified deferred compensation plan, and therefore do not make the employee an “active” participant in a qualified retirement plan. The 401(k) is a qualified plan and the 403(b) and SEP are ‘wannabe’ be plans that would make the employee an “active” participant.

107
Q

June and Bud, both 40 years old, are not covered by a qualified retirement plan. Bud, trying to maximize their IRA deduction, put $12,000 into an IRA with June as the beneficiary on December 15 of the current year. What best describes the result of this transaction?

June and Bud receive a tax deduction for the entire $12,000 because both spouses are eligible to contribute $6,000 to the IRA.
Bud receives a tax deduction for $6,000 and a 6% penalty for over-contribution on the other $6,000.
Next year Bud will receive a $6,000 deduction, in addition to the $6,000 deduction for this year.
Bud receives a tax deduction for $6,000 and is considered to have made a non-deductible contribution of the other $6,000.

A

Bud receives a tax deduction for $6,000 and a 6% penalty for over-contribution on the other $6,000.
- This question indicates an IRA in only Bud’s name. Maximum contribution is $6,000 plus any applicable catch-up provisions. Amounts contributed over that level are considered excess contributions and subject to a 6% penalty until taken out. The 6% penalty could have been avoided if the excess contribution was withdrawn prior to the original filing deadline without extension.

108
Q

Nicole has adjusted gross income for the year of $126,000, has just retired, and wants to roll-over her employer sponsored profit-sharing plan into a Roth IRA. The profit-sharing plan is currently worth $291,000 made up entirely of employer contributions. Which of the following is not true?

Nicole may rollover the entire amount of her profit-sharing plan.
Nicole must include the converted amount in taxable income upon conversion.
Distributions of any converted amount will be tax free.
Penalties will always be due on the converted amounts withdrawn for purposes other than the exceptions to 72(t).

A

Penalties will always be due on the converted amounts withdrawn for purposes other than the exceptions to 72(t).
- (A) Beginning in 2010 there is no AGI phase-out associated with conversions to Roth IRAs so she can rollover as little or as much as she wants. (B) The converted amount will be included in taxable income in the year of conversion. (C) Distributions of any of the converted amounts will always be tax-free; however penalties may be due under certain circumstances. (D) Penalties on converted amounts are applicable within the first 5 years from conversion only.

109
Q

Cher, who just turned 57 years old, took early retirement so she could spend more time with her three grandchildren and to work on her golf game. She has the following accounts: 401(k) Roth account - she has a balance of $100,000. She only worked for the company for four years and contributed $15,000 each year to the Roth account. The company never contributed anything to her account. Roth IRA - she has a balance of $80,000. She first established the account by converting her traditional IRA ($50,000 all pretax) to the Roth IRA 4 years ago and has contributed $5,000 each of the last 4 years. Cher decided that she would take a distribution of half of each account ($50,000 from the Roth 401(k) and $40,000 from the Roth IRA) for the purpose of purchasing a Porsche Cayenne, which of course would be used to carry her new Ping golf clubs. Which of the following is correct regarding the tax treatment of her distributions?

No tax, no penalty on either distribution.
Taxation on $20,000 from the 401(k) Roth and a penalty on $20,000 from the Roth IRA.
No taxation on the distribution from the 401(k) Roth, but income and penalty on $20,000 from the Roth IRA.
Penalty of $2,000 on the Roth distribution and taxation and penalty on $20,000 of the Roth 401(k) distribution.

A

Taxation on $20,000 from the 401(k) Roth and a penalty on $20,000 from the Roth IRA.
- Neither distribution is qualified. Non-qualified distributions from a Roth account consist of basis and earnings on a pro rata basis. Therefore, 60% of the Roth account distribution is return of basis. The remaining 40% or $20,000 is subject to income tax. Because the distribution is from a qualified plan and she has separated after the attainment of age 55, there is no penalty. Non-qualified distributions from a Roth IRA come out in the order of contributions, conversions and then earnings. The first $20,000 is not subject to income tax or penalty because it is from contributions. The second $20,000 is from conversions, which have been subject to taxation. However, because she rolled them over within the last five years, she will have a penalty and there is no exception.

110
Q

Ernest converted his Traditional IRA to a Roth IRA on Dec 15, 2014. He was 35 years of age at the time and had never made a contribution to a Roth IRA. The conversion was in the amount of $60,000 ($10,000 of contributions and $50,000 of earnings). Over the years he has also made $15,000 in contributions. On May 15, 2018 he withdrew the entire account balance of $100,000 to pay for a 1 year trip around the world. Which of the following statements is true?

$25,000 of the distribution will be subject to income tax and $85,000 of the distribution will be subject to the 10% early withdrawal penalty.
$25,000 of the distribution will be subject to income tax and the 10% early withdrawal penalty.
Some of the distribution will be taxable but the entire distribution will be subject to the 10% early withdrawal penalty.
None of the distribution will be taxable nor will it be subject to the 10% early withdrawal penalty.

A

$25,000 of the distribution will be subject to income tax and $85,000 of the distribution will be subject to the 10% early withdrawal penalty.

  • Roth distributions are tax free if they are made after 5 years and because of 1)Death, 2)Disability, 3) 59.5 years of age, and 4)First time home purchase. He does not meet the five year holding period or one of the exceptions. His distribution does not received tax free treatment.
  • The treatment for a non-qualifying distribution allows the distributions to be made from basis first, then conversions, then earnings. His basis will be tax free.
  • The conversion is also tax free since we paid tax at the time of the conversion on those earnings.
  • The remaining earnings since establishment of the Roth are $25,000 (100,000 - $15,000 in basis - $60,000 in conversions) and will be taxed.
  • The 10% penalty does apply to this distribution since he does not qualify for any of the exceptions to the penalty. The contributions escapes penalty but the conversions and earnings of $85,000 are subject to the 10% early withdrawal penalty.
  • Remember that in order for the conversions to escape the 10% early withdrawal penalty the distribution must occur after a 5 year holding period beginning Jan 1 in the year of conversion or meet one of the 10% early withdrawal exceptions.
111
Q

In order to deduct a contribution to an IRA, which of the following requirements must be met?

I. An individual must have earned income, either personally or jointly from a spouse.
II. Must not be an active participant in an employer-sponsored qualified plan.
III. Must be under the age of 70 1/2.
IV. Must make contributions during the tax year or up to the date of filing the federal tax return for the tax year, including extensions.

I and II only.
I and III only.
II and III only.
II and IV only.

A

I and III only.
- In 2019, contributions are limited to the lesser of 100% of earned income or $6,000 or $7,000 if age 50 or over. Deductions may be taken even if an active participant, so being a non-participant is not a requirement. The IRA holder must be under age 70 1/2 to make contributions into an IRA. Contributions must be made prior to April 15 (or the mandated filing date for the year.) No extension to make the contribution is allowed after that date, even though an extension to file the return is granted.

112
Q

Your client, Jill, age 49 in the current year, has earned income of $3,500. Her spouse, James, is retired. They also receive $30,000 per year from his pension and $100,000 in installment income from the sale of a business. Jill put $1,000 into a traditional IRA. They each decide to invest in a Roth IRA. What is the most Jill can put into her personal Roth IRA?

$500
$2,500
$3,500
$5,000

A

$2,500
- Jill is limited to a maximum IRA contribution of $6,000 (2019) or 100% of earned income ($3,500) since their joint AGI is below the Roth IRA phase-out threshold. She has already contributed $1,000 to traditional IRA, so maximum available for the Roth is $2,500 ($3,500 earned less $1,000).

113
Q

Kyle had contributed $20,000 in nondeductible contributions to his traditional IRA over the years. This year the account balance was $52,000 and he made a withdrawal of $5,000. What amount is reported on Kyle’s Form 1040?

$5,000 only
$1,923 only
$3,077 only
Both $5,000 and $3,077

A

Both $5,000 and $3,077

  • On Form 1040 Kyle will report the total distribution of $5,000 and the taxable amount of the distribution of $3,077 calculated as $32,000 ÷ $52,000 × $5,000.
  • account balance = $52,000
  • non-deductible contributions = $20,000 (not taxed at distribution)
  • $32,000 would be taxable.
  • Because there is both taxable and non-taxable money, each distribution is a pro-rata distribution of both.
  • 32k/52k = .6154
  • 5,000 × 61.54% = 3,076.92
114
Q

Ginger is a 75 year old retired actress. Although she enjoyed a lucrative career, her decline in health has prevented her from working for the last few years. She is currently contemplating contributing to a Roth or Traditional IRA. Which of the following best describes her options?

She can’t contribute to a Traditional IRA because she is too old, but she can contribute to a Roth IRA.
She can’t contribute to a Roth IRA because she is too old, but she can contribute to a Traditional IRA.
She can contribute to either a Traditional IRA or a ROTH IRA but is not entitled to a deduction.
She can’t contribute to either a Traditional or Roth IRA.

A

She can’t contribute to either a Traditional or Roth IRA.
- There is no mention of earned income and “her health has prevented her from working.” Therefore, she can’t contribute to either one. You cannot assume earned income, and in fact, this question indicates that she hasn’t worked at all for several years. If she did have earned income then she would be over the age limit to contribute to a Traditional IRA (70.5 years) but there is no age limit for Roth IRAs.

115
Q

A client, age 54 and single, chose early retirement and is receiving from his previous employer’s qualified pension plan a monthly pension of $750. This year, the client elects to work for a small company and will receive $25,000 in annual compensation. This company does cover him under a qualified pension plan. The client wants to contribute the maximum deductible amount to an Individual Retirement Account (IRA). The amount of the IRA contribution that he can deduct from his gross income in 2019 is:

$0
$4,000
$6,000
$7,000

A

$7,000
- Taxpayers who work and have earned income may contribute the lesser of their earned income or $6,000 to an IRA (2019). If the taxpayer is currently covered under an employer’s qualified plan, they may still contribute to the plan but (depending on their earnings from the employer with whom they are covered) may render the contributions non-deductible. Since the taxpayer earned in excess of $6,000, he may contribute the full $6,000, and age 50 or over, he may add an additional $1,000, as a catch up for a total of $7,000. His total income is only $34,000, which is much lower than the phaseout limits. Therefore, he can also deduct the $7,000 contribution.

116
Q

Martha has AGI of $1,000,000 (which is all comprised of earned income). She is single and age 45. She participates in her employer’s SIMPLE plan. Which of the following statements is true?

She can contribute $6,000 to a Traditional IRA and deduct all $6,000.
She can contribute $7,000 to a Traditional IRA and deduct all $7,000.
She can contribute $6,000 to a Traditional IRA and deduct $0.
She can contribute $7,000 to a Traditional IRA and deduct $0.

A

She can contribute $6,000 to a Traditional IRA and deduct $0.
- Participating in a SIMPLE plan is considered being an “Active Participant.” She can contribute $6,000 (2019) to a Traditional IRA but cannot deduct any since she is above the AGI limitation for a single active participant ($64,000 - $74,000) (2019). She is unable to contribute to a Roth IRA because she is above the AGI limitation of $122,000 - $137,000 (2019). Because she is not 50 or older she is not allowed to make the $1,000 (2019) catch up contribution.

117
Q

Which of the following statements apply to distributions made from Individual Retirement Accounts (IRA)?

I. Distributions to the IRA owner must begin by April 1 of the year following the year in which the owner reaches age 70 1/2.
II. If funds in a rollover IRA (originated in an employer-sponsored qualified retirement plan) are not “tainted” with other contributions, the distribution may be eligible for 5-year forward averaging tax treatment.
III. After the owner’s death, the entire amount remaining in the IRA is included in the owner’s gross estate for federal estate tax purposes.
IV. Distributions taken prior to age 59 1/2 may be exempt from penalty only if the owner separated from service after age 55 and the original plan document allowed early retirement at age 55.

I and II only.
I and III only.
II and IV only.
I, III and IV only.

A

I and III only.
- Statement “II” is incorrect because funds distributed from an IRA are always treated as ordinary income, regardless of source and 5 year forward averaging is no longer available for any distribution. Statement “IV” is incorrect because all distributions from an IRA not meeting the statutory exemptions are subject to the premature distribution penalty, regardless of source.

118
Q

Sean, age 75 and Jaclyn, age 45, are married filing joint and have AGI of $200,000 (which is all comprised of earned income). Neither are active participants in a qualified plan. If they contributed the maximum allowed by law to their Traditional IRAs what is their available Above the Line Deduction for these contributions?

$0
$6,000
$12,000
$13,000

A

$6,000
- Since they are not active participants there is no AGI limitation. However, Sean is above 70.5 and therefore is unable to make a contribution to a Traditional IRA. Jaclyn is able to make a $6,000 (2019) deductible contribution. She does not qualify for the catch up because she is not 50 or older.

119
Q

Donald and Daisy are married and file jointly. They are both age 42, both work, and their combined AGI is $115,000. This year Donald’s profit sharing account earned over $5,000. Neither he nor the company made any contributions and there were no forfeitures. Daisy declined to participate in her company’s defined benefit plan because she wants to contribute to and manage her own retirement money. (Her benefit at age 65 under the plan is $240 a month.) How much of their $12,000 IRA contribution can they deduct? Assume that $6,000 is contributed to each account.

$6,000
$8,400
$9,600
$12,000

A

$8,400
- Daisy is an active participant. She cannot opt out of a defined benefit plan.

Reduction = 6,000 × [(115,000-103,000) ÷ 20,000]

Reduction = 3,600

$6,000 - $3,600 = $2,400 deductible contribution.

Donald is not active in the current year so he is eligible for a spousal IRA of $6,000.

$6,000 + $2,400= $8,400

Formula: Contribution × [(your AGI – the bottom of the phase out range) divided by the amount of the range (123,000-103,000 is where the 20k comes from)]

120
Q

Deepak made a contribution to his Roth IRA on April 15, 2017 for 2016. He was 58 years of age at the time and decided it was time he made his first contribution to a Roth IRA. Over the years he has made $30,000 in contributions. On May 15, 2019 the entire account balance was $50,000 and he took out $45,000 to pay for his wedding and honeymoon. Which of the following statements is true?

He will not include anything in income and will not be subject to the 10% early withdrawal penalty.
He will include $15,000 in income and will be subject to the 10% early withdrawal penalty on $15,000.
He will include $15,000 in income but will not be subject to the 10% early withdrawal penalty.
He will include $20,000 in income but will not be subject to the 10% early withdrawal penalty.

A

He will include $15,000 in income but will not be subject to the 10% early withdrawal penalty.
- Roth distributions are tax free if they are made after 5 years and because of

1) Death, 2) Disability, 3) 59.5 years of age, and 4) First time home purchase. He does meet a qualifying reason because he is over 59.5 in 2019 if he was 58 in 2017. However, he did not meet the 5 year holding period. He only has about 4.5 years. His distribution does not received tax free treatment. The treatment for a non-qualifying distribution allows the distributions to be made from basis first, then conversions, then earnings. His basis will be tax free, leaving only the earnings as taxable income. Since he did not take the entire account balance he will only be subject to tax on the $15,000 of earning withdrawn. The 10% penalty does not apply to this distribution since he qualifies for the 59.5 exception to the penalty.

121
Q

Your Uncle Ben began receiving required minimum distributions from his IRA in 2005 and has died leaving a balance in his IRA. He has named you as beneficiary (non-spouse beneficiary). Which of the following identifies your minimum distribution rule?

Distributions may be made over your life expectancy (fixed) beginning by December 31st in the year of death.
Distributions may only be made over a five-year period.
Distributions must be made over Uncle Ben’s life expectancy (recalculated) with the first distribution made by December 31st of the year following death.
Distributions may be made over your life expectancy (fixed) beginning by December 31st of the year following death.

A

Distributions may be made over your life expectancy (fixed) beginning by December 31st of the year following death.
- Note that for a non-spouse beneficiary, the Single Life Table is entered once in the year following death based on the age of the beneficiary and then that number declines by a factor of 1 each year to determine the divisor for the following distributions. The non-spouse beneficiary could make a direct transfer to an “inherited IRA”. Answer “A” is incorrect because the distributions to the beneficiary are based on the beneficiary’s life expectancy beginning in the year following death. Do point out that the distribution in the year of death is still based on the owner’s life. Answer “B” is incorrect because the five-year rule is NOT the only choice. Answer “C” is incorrect because a non-spouse beneficiary would not receive distributions based on the owner’s life expectancy nor would a post-death non-spouse distribution be recalculated.

122
Q

A 56-year-old client becomes unemployed due to a disability. The client tells a CFP® professional that he hopes to go back to work eventually, but is not sure when that might be. Until then, he needs to generate replacement income. His only available asset is his traditional 401(k) plan. What is the best way for the client to replace his income (CFP® Certification Examination, released 8/2012)?

Directly from the 401(k) plan
From a rollover IRA, using Rule 72(t)
From a rollover annuity, using substantially equal payments
From a brokerage account, using net unrealized appreciation (NUA)

A

Directly from the 401(k) plan

- The 401(k) plan is penalty-free and will not require continued payments if the client goes back to work.

123
Q

A client becomes age 70 1/2 on October 1 of this year and must receive a minimum distribution from his IRA account. The value of the account at the beginning of the current year was $53,000. His spouse, age 63, is the beneficiary of the IRA account. The applicable divisor for his distribution is 26.5. If the client takes a $1,000 distribution by April 1, what is the tax penalty, if any?

$0
$100
$150
$500

A

$500
- The minimum distribution is calculated by taking the account value at the beginning of the year ($53,000 in this case) and dividing by the applicable divisor (life expectancy 26.5 years in this example). The resulting figure is the minimum distribution ($2,000). Because the amount distributed was $1,000 less than required under minimum distribution rules, a 50% penalty applies. 50% of the undistributed minimum distribution is $500.

124
Q

Chris Barry, 59-years old, has been offered early retirement with an option of a two-year consulting contract. He has been a participant for the past 20 years in both the company defined benefit plan and defined contribution plan. His account balance is $120,000 in the profit-sharing plan and the present value of accrued benefit of the defined benefit plan is $240,000. Both provide for a lump sum distribution. Which of the following option(s) is/are available under the lump sum distribution rules?

Elect ten-year averaging on both plans.
Roll over the taxable portions of both plans to an IRA.
Elect long-term capital gains treatment on the DB plan.
Elect five-year averaging on both plans.
I, II and III only.
I and II only.
II only.
IV only.
A

II only.
- Statement “I” is incorrect because he is not old enough for ten-year averaging. Statement “II” is correct because the taxable portion of any lump sum distribution may be rolled over into an IRA. Statement “III” is incorrect because he is not old enough to qualify for pre-74 capital gain treatment nor does he even have any actual pre-74 capital gain in the plan as he has only been in the plan for the last 20 years. Statement “IV” is incorrect because five-year averaging was repealed in 1999.

125
Q

Which of the following will be subject to a 10% early withdrawal penalty?

Sylvia, age 56, retired from Marshall Corporation. She takes a $125,000 distribution from the Marshall Corporation Defined Contribution Retirement Plan to pay for living expenses until she is eligible for Social Security.
Terry quits Shoe Shine Company at age 48. He begins taking equal distributions over his life expectancy from his qualified plan after separating from service. The annual distribution is $2,000.
Kevin leaves Hedwig University at age 50. He takes a $1,000,000 distribution from his defined contribution pension plan. Six weeks after receiving the $800,000 check (net of 20% withholding), Kevin deposits $1,000,000 into a new IRA account.
Edward, age 40, takes a $40,000 distribution from his profit-sharing plan to pay for his son’s college tuition.

A

Edward, age 40, takes a $40,000 distribution from his profit-sharing plan to pay for his son’s college tuition.
- There is no provision for a distribution without penalty under this circumstance. Edward is only 40 and education withdrawals are allowed in IRAs, not from qualified plans.

126
Q

Which of the following is/are elements of an effective waiver for a preretirement survivor annuity?

The waiver must be signed within six months of death.
The waiver must be signed only by a plan participant.
The waiver must be notarized or signed by a plan official.

III only
I and II only
II and III only
I, II and III

A

III only
- Both the plan participant and the nonparticipant spouse must sign the waiver. Statement “III” is correct. Statement “I” is unfounded.

127
Q

A premature distribution from a qualified retirement plan is allowed at age 52 without a 10% penalty tax when a participant:

I. Becomes obligated for payment of plan benefits to an alternate payer under a qualified domestic relations order (QDRO).
II. Separates from service and takes an accepted form of systematic payment.
III. Remains with current employer but elects to take systematic payments over the life of the participant and spouse.

I only.
III only.
I and II only.
I, II and III.

A

I and II only.
- Distributions under a QDRO are not taxable to the taxpayer actually making the disbursement from his/her account. IRC 72(t) allows Substantially Equal Payment Plans (SEPP) to escape the 10% penalty as long as the payments continue for the longer of 5 years or until age 59 1/2. No in-service withdrawals are exempted from the 10% early withdrawal penalty.

128
Q

Jacque’s wife just lost her job and they had a death in the family. Jacque is planning on taking a hardship withdrawal from his 401(k) plan to pay for living expenses and funeral costs. Which of the following is correct regarding hardship withdrawals?

Hardship withdrawals can be taken even if there is another source of funds that the taxpayer could use to pay for the hardship.
Hardship withdrawals are beneficial because although they are taxable, they are not subject to the early withdrawal penalty.
Hardship withdrawals can be taken from elective deferral amounts or vested employer contributions.
Unless the employer has actual knowledge to the contrary, the employer may rely on the written representation of the employee to satisfy the need of heavy financial need.

A

Unless the employer has actual knowledge to the contrary, the employer may rely on the written representation of the employee to satisfy the need of heavy financial need.
- Answer a is not correct as there must not be another source of funds. Answer b is not correct as they are generally subject to a penalty unless there is an exception under IRC 72(t). Answer c is not correct as a hardship distribution can only be taken from employee deferrals.

129
Q

David Lee, age 63, was a participant in a stock bonus plan sponsored by VH, Inc., a closely held corporation. David’s account was credited with contributions in shares of VH stock to the stock bonus plan and VH Inc. deducted $80,000 over his career at VH. The value of the stock in the account today is worth $1 million. David takes a distribution (year 1) of one-half of the VH stock in his stock bonus plan account valued at a fair market value of $500,000. If David sells the stock for $600,000 nine months after receiving the distribution (year 2), then which of the following statements are true?

David will have ordinary income of $80,000 in year 1 and capital gain of $520,000 in year 2.
David will have ordinary income of $40,000 in year 1 and capital gain of $560,000 in year 2.
David will have ordinary income of $460,000 in year 1 and capital gain of $100,000 in year 2.
David will have ordinary income of $500,000 in year 1 and capital gain of $100,000 in year 2.

A

David will have ordinary income of $500,000 in year 1 and capital gain of $100,000 in year 2.
- NUA treatment is not applicable because David did not take a lump sum distribution of stock from the plan. Therefore, the distribution is treated as ordinary income.

130
Q

Loans from a qualified plan are considered prohibited transactions unless the following requirements are met:

I. There must be a reasonable rate of interest.
II. There is a dollar limitation of the lesser of 50% of the account balance or $12,000.
III. The term of the loan is 10 years unless for primary residence.
IV. There is sufficient security.

I and IV only.
I, III and IV only.
II, III and IV only.
I, II and IV only.

A

I and IV only.
- Limit on loans is the lesser of 50% of the account value or $50,000 for a period of five years, unless for a primary residence.

131
Q

If a stock option is vested when it is received, and has a readily ascertainable value it is:

Assigned that value for taxation purposes.
Taxable when the stock is sold.
Taxable as soon as it is exercised.
Immediately taxable.

A

Taxable as soon as it is exercised.
Immediately taxable.
- Vested options are taxable based on the value of the option to the extent the Fair Market Value exceeds the option price.

132
Q

Which of the following is false regarding incentive stock options?

I. No regular taxable income will be recognized by the employee when the qualified option is granted or exercised.
II. The income from sale of the qualified option will always be taxed as capital gains when the stock is sold.
III. The income from sale of the qualified option will be taxed as ordinary income regardless of when the stock is sold.
IV. The employer will not be able to deduct the bargain element of the option as an expense under any circumstance.
V. For favorable tax treatment the stock must be held two years from grant and one year after exercise.

II and IV only.
II, III and IV only.
III only.
I, II and V only.

A

II, III and IV only.
- Only Statements “I” and “V” are true. The rest are all false. In Statement “II”, be careful of “always”! In Statement “III,” if held longer than one year, they receive capital gains treatment. In Statement “IV,” the bargain element will be deductible if the sale is a disqualifying disposition.

133
Q

Jacinth is an executive at Papers Unlimited. As part of her compensation she has a restricted stock plan that allows her to receive 2,000 shares of stock after she completes of 5 years of service. At the time of grant the stock was trading at $4 per share. She did not make the 83b election. She met the vesting requirement 8 months ago when the stock was trading at $28. She has decided to sell her stock. All of the following are true, except:

If she sells the stock today for $3 per share she would have an ordinary loss of $50,000.
If she sells the stock today for $15 per share she will recognize a capital loss of $26,000.
If she sells the stock today for $28 per share then she will not recognize any gain or loss.
If she sells the stock today for $38 per share then she will recognize $20,000 in short term capital gains.

A

If she sells the stock today for $3 per share she would have an ordinary loss of $50,000.
- When she met the vesting period she would have recognized W-2 income of $56,000 (2,000 × $28).

If she sold the stock at $3 then she would recognize a capital loss of $50,000 ($56,000 basis - $6,000 sale price).

If she sold the stock at $15 then she would recognize a capital loss of $26,000 ($56,000 basis - $30,000 sale price).

If she sold the stock at $28 then she would not recognize any gain or loss ($56,000 sale price - $56,000 basis).

If she sold the stock at $38 then she would recognize a short term gain of $20,000 ($76,000 sale price - $56,000 basis).

134
Q

Meredith is an executive at Papers Unlimited. As part of her compensation she has a restricted stock plan that allows her to receive 1,000 shares of stock after she completes of 5 years of service. At the time of grant the stock was trading at $2 per share. She made a proper 83b election. She met the vesting requirement 6 months ago when the stock was trading at $35. She has decided to sell her stock. Which of the following is true?

If she sells the stock today for $1 per share she is not entitled to a loss.
If she sells the stock today for $15 per share she will recognize $13,000 in long term capital gain.
If she sells the stock today for $28 per share then she will recognize $26,000 in short term capital gains.
If she sells the stock today for $38 per share then she will recognize $3,000 in short term capital gains.

A

If she sells the stock today for $15 per share she will recognize $13,000 in long term capital gain.
- When she made the 83b election she would have recognized W-2 income of $2,000 (1,000 × $2). Her holding period would have started at the date of grant. When she met the vesting period she would not have recognized anything since she made the 83b election. If she sold the stock at $1 then she would have had a loss of $1,000 ($2,000 basis - $1,000 sale price). If she sold the stock for $15 then she would have long term capital gain of $13,000 (1,000 × ($15 - $2)). If she sold the stock for $28 then she would have long term capital gain of $26,000 (1,000 × ($28 - $2)). If she sold the stock for $38 then she would have long term capital gain of $36,000 (1,000 × ($38 - $2)).

135
Q

Which of the following statements are true in regards to Section 457 plans?

I. Eligible plan sponsors include non-profit organizations, churches, and governmental entities.
II. In-service distributions after age 59 1/2 are allowed in a 457 plan.
III. Salary deferrals are subject to Social Security, Medicare, and Federal unemployment tax in the year of the deferral.
IV. Assets of the plans for non-government entities are subject to the claims of the sponsor’s general creditors.

I and III only.
III and IV only.
I, II and IV only.
II, III and IV only.

A

III and IV only.

- Churches are not qualifying sponsors of 457 plans. In-service distributions are not allowed until age 70 1/2.

136
Q

Which of the following statements accurately reflect the characteristics of a Section 457 plan?

I. Benefits taken as periodic payments are treated as ordinary income for taxation.
II. Lump-sum distributions are eligible for 5-year and/or 10-year averaging.
III. Deferred amounts are subject to Social Security and Medicare taxes at the later of: performance of services or employee becomes vested in the benefits.
IV. Income tax withholding is not required until funds are actually received, not constructively received.
V. Cannot exceed the smaller of $19,000 or 100% includible compensation.

I, III and V only.
II, IV and V only.
I, II, IV and V only.
I, II, III, IV and V.

A

I, III and V only.
- There are no special tax advantages provided for 457 plans distributed in a lump-sum. Income tax withholding is required once the benefits are constructively received, even if not actually received.

137
Q

Randal was just hired by Chastain, Inc., which sponsors a defined benefit plan. After speaking with the benefits coordinator, Randal is still confused regarding eligibility and coverage for the plan. Which of the following is correct?

The plan could provide that employees be age 26 and have 1 year of service before becoming eligible if upon entering the plan, the employee is fully (100%) vested.
The plan may not cover Randal due to his position in the company, even if Randal meets the eligibility requirements.
Part-time employees, those that work less than 1,000 hours within a twelve-month period, are always excluded from defined benefit plans.
Generally, employees begin accruing benefits as soon as they meet the eligibility requirements.

A

The plan may not cover Randal due to his position in the company, even if Randal meets the eligibility requirements.
- Choice a is not correct because the general eligibility is age 21, not 26. Choice c is not correct because a plan could cover part time employees, but will generally not. Choice d is not correct because employees become part of a plan only as early as at the next available entrance date after meeting the eligibility requirements.

138
Q

Tara is a participant in Kean Co.’s defined benefit plan and standard 401(k) plan. Tara, who is a mid-level manager, is 44 years old and earns $100,000. She has five years of service for purposes of the plans and has worked at Kean for five years. The plan provides a benefit of 2% for each year of service. Both plans have the least generous graduated vesting schedule possible. Almost eighty percent of the accrued benefits in the defined benefit plan are attributable to the rank and file employees, and not the owners. According to the actuary, Tara’s accrued benefit in the defined benefit plan is $10,000. Over the last five years, Tara has deferred a total of $30,000 from her salary, which has grown to $40,000. In addition, Kean has matched these contributions with $15,000, which is now worth $20,000. If Tara were to leave today, how much could she rollover into a new employer’s plan?

$70,000
$64,000
$62,000
$56,000

A

$62,000

  • The DB plan is not top heavy. Therefore, the vesting for the DB plan is a 7 year graded schedule and he is 60% vested in the $10,000. She is 100% vested in the $40,000, but only 80% vested in the employer matching contribution.
  • This question requires you to know the vesting schedules, which are not provided on the exam.
  • For DB either 5 year cliff or 7 year graded, for DC 3 year cliff or 6 year graded.
  • DB plan has 10,000 and is 60% vested = 6,000
  • DC Employee contribution is $40,000 (100% vested)
  • DC Employer match is 20,000 and is 80% vested = 16,000
  • Total vested is $62,000
139
Q

Jerome is covered under his employer’s money purchase pension plan. Several things happened in the current year. Which of the following would increase the company contributions for the current year?

The company gave all key employees a 5% raise and all non-key employees a 3% raise.
One of the key employees retired.
The company had two employees terminate who forfeited a total of $10,000. The forfeitures were allocated to the remaining participants.
The equity market declined and all account balances declined by at least 2%.

A

The company gave all key employees a 5% raise and all non-key employees a 3% raise.
- If the company gave everyone a raise then that would increase the company’s contributions. If a key employee retired and two employees left that would decrease the company’s contributions. Since the forfeiture allocations were allocated to the participants they would have no effect of the company’s contributions. In a money-purchase pension plan the investment risk is on the employees and thus an increase or decrease in the investments has no impact on contributions.

140
Q

Carolyn Smart wanted to volunteer full-time and decided to retire from Lotsa Cash Corporation at the age 57, after 15 years of service. She requested a total distribution of her account in the Lotsa Cash Corporation’s profit sharing plan and received a check, made payable to her. Her account balance was $60,000 on her final day of employment. Which of the following statements describe the consequences of this distribution?

I. Eligible for 10 year forward averaging
II. Subject to 10% penalty
III. Eligible for Rollover
IV. Subject to mandatory 20% withholding
V. Exempt from the 10% early withdrawal penalty

I, II and III only
II, III and IV only
III, IV and V only
III and IV only

A

III, IV and V only
- Statements II and V cannot co-exist. She is not old enough for statement I. She is not subject to 10% penalty because this is a qualified plan and she is over 55 (10% early withdrawal is waived for separation of service after age 55). Distributions from qualified plans are subject to mandatory 20% withholding.

141
Q

Myron has a life insurance policy in his qualified plan at work. He has come to you for advice about retirement and other financial planning needs. Which of the following is not correct about the life insurance in a qualified plan?

He will be subject to income only if the policy in his qualified plan is a cash value type policy.
The policy will be included in his gross estate if he were to die while still working.
Part of the proceeds could be taxable to his beneficiary if it is a cash value policy.
When he distributes the policy from his plan at retirement, he can convert it to an annuity within 60 days to avoid taxation.

A

He will be subject to income only if the policy in his qualified plan is a cash value type policy.
- Statements b, c, and d are correct. Statement a is false because all life insurance in qualified plans is subject to income when purchased, regardless of the type.

142
Q

Which of the following statements regarding determination letters for qualified plans is true?

When a qualified plan is created, the plan sponsor must request a determination letter from the IRS.
An employer who adopts a prototype plan must request a determination letter from the IRS.
If a qualified plan is amended, the plan sponsor must request a determination letter from the Department of Labor.
A qualified plan which receives a favorable determination letter from the IRS may still be disqualified at a later date.

A

A qualified plan which receives a favorable determination letter from the IRS may still be disqualified at a later date.
- Determination letters are issued by the IRS at the request of the plan sponsor. The plan sponsor is not required to request a determination letter. Even if the determination letter is requested and approved, the IRS may still disqualify the plan.

143
Q

According to ERISA, which of the following is/are required to be distributed annually to defined benefit plan participants or beneficiaries?

I. Individual Benefit Statement.
II. The plan’s summary annual report.
III. A detailed descriptive list of investments in the plan’s fund.
IV. Terminating employee’s benefit statement.

I, II and IV only.
I and II only.
II and IV only.
III and IV only.

A

II and IV only.
- Individual Benefit Statements are not required annually for defined benefit plans. They are however, required at least once every three years. Alternatively, defined benefit plans can satisfy this requierment if at least once each year the administrator provides notice of the availability of the pension benefit statement and the ways to obtain such statement. In addition, the plan administrator of a defined benefit plan must furnish a benefit statement to a participant or beneficiary upon written request, limited to one request during any 12-month period. There are no individual accounts in a defined benefit plan, so a specific listing of invested assets is not required.

144
Q

Which of the following is a correct definition of qualified plan tests for eligibility?

Ratio percentage test - Plan must cover a percentage of non-highly compensated employees that is at least 60% of the percentage of highly compensated employees covered.
Average benefits test - Plan must benefit a non-discriminatory employee class with benefits of at least 70% of the benefit provided key employees.
50/40 test - 50% of all employees must participate, or a minimum of 40 employees, or 2 employees out of 3 if there are only 3 employees.
Plans cannot require more than 1 year of service, and an age higher than 21. The plan can require a 2-year waiting period if there is immediate 100% vesting in the plan.

A

Plans cannot require more than 1 year of service, and an age higher than 21. The plan can require a 2-year waiting period if there is immediate 100% vesting in the plan.
- Answer “A” should read 70%. Answer “B” should read “highly compensated.” Answer “C” is reversed with 50 employees or 40% of employees with a minimum of 2 out of 3 employees (unless there is only one employee in which case only 1 participant is required).

145
Q

Ballistic Laser Operated Weapons company (BLOW) is a defense contractor who develops innovative weapons involving laser-guided systems. The company has maintained a defined benefit plan and a money purchase pension plan for many years. The current benefit formula for the defined benefit plan equals 3% times years of service times the average of the last three years of salary, limited to a maximum benefit of 70%. The money purchase pension plan calls for a 6% contribution for all employees who are covered under the plan. BLOW has been experiencing financial difficulties due to changes in the industry and from competitors and alternative technologies. Based on these challenges, the company is considering changing the benefits under the plans. Which of the following changes would not be permitted under the anti-cutback rules?

Changing the benefit accrual for the defined benefit plan from 3% per year to 2% per year for future years.
Reducing the money purchase pension plan contribution from 6% to 3% for future years.
Decreasing the maximum benefit under the defined benefit plan from 70% to 50% for all future retirees.
Switching the vesting for the money purchase pension plan from 3 year cliff to 2 to 6 graduated vestsing.

A

Decreasing the maximum benefit under the defined benefit plan from 70% to 50% for all future retirees.
- The anti-cutback rules state that you cannot “cutback” benefits that have been accrued to date. Choice a and b affect future benefits. Choice c will more than likely impact current employees who may have accrued 70% benefits, but who have not yet retired. The change would result in a reduction in benefits and is not permitted. Choice d does is a permitted change and would not result in a reduciton in current vesting.

146
Q

To retain its qualified status, a retirement plan must:

I. Have pre-death and post-death distributions.
II. Stipulate rules under what circumstances employee contributions are forfeited.
III. Be intended to be permanent.
IV. Be established by the employer.

I and II only.
II, III and IV only.
I, III and IV only.
I, II, III and IV.

A

I, III and IV only.

- Employee contributions must be vested and cannot be required to be forfeited.

147
Q

Which of the following employees can be excluded from participation in a qualified plan?

Age 22 with three years of service.
Employee (with 13 months service) of 401(k) plan sponsor.
Previously eligible employee terminated from service with 501 hours during plan year.
Collective bargain covered employee of 2 years.

A

Collective bargain covered employee of 2 years.
- Maximum exclusions are: age 21, three years of service for a SEP, 2 years of service for all other plans except the 401(k) which has a maximum exclusion period of one year. Employees covered under a pension plan in a collective bargaining agreement can always be excluded from participation in the plan because they are already receiving pension contributions through the union plan.

148
Q

Which of the following tasks are the primary responsibilities of a plan trustee?

I. Determining which employees are eligible for participation in the plan, vesting schedule, and plan benefits.
II. Preparing, distributing, and filing reports and records as required by ERISA.
III. Investing the plan assets in a “prudent” manner.
IV. Monitoring and reviewing the performance of plan assets.

I and III only.
I and II only.
II and IV only.
III and IV only.

A

III and IV only.

- The duties explained in Statements “I” and “II” are responsibilities of the plan administrator.

149
Q

Safe harbor requirements to exclude leased employees from an employer’s retirement plan include all but the following:

The leasing company must maintain a money-purchase plan with a contribution rate of 10%.
The retirement plan of the leasing company may be integrated.
The leasing company’s plan must provide immediate vesting.
Safe harbor can be used until leased employees constitute 20% of the non-highly compensated work force.

A

The retirement plan of the leasing company may be integrated.
- Under the safe-harbor leasing rules the plan must provide a 10%, non-integrated money purchase plan with immediate vesting. No more than 20% of the employer’s non-highly compensated employees may be leased to qualify for the safe harbor rules.

150
Q

The maximum service requirement that a profit sharing plan may impose as a condition of participation is:

1 year with semi-annual entry dates (1.5 years).
1 year with quarterly entry dates (1.25 years).
6 months.
2 years.

A

2 years.

- This requires “immediate vesting.”

151
Q

Complex Corporation is ready to adopt a profit sharing plan for eligible employees. Which of the following groups would have to be considered in meeting the statutory coverage and participation tests?

I. Employees of Simple Corporation, in which Complex owns 85% of the stock.
II. Employees of Universal Corporation, in which Complex owns 55% of the stock.
III. Rank and file workers at Complex who are union members with a contract that provides retirement benefits as a result of good-faith collective bargaining.
IV. Employees who are leased and covered by the leasing corporation’s profit sharing plan.

I only
I and II
I and IV
II and III

A

I and IV
- Simple must be considered because Complex owns more than 80% and the leased employees must be considered because their leasing company’s is not a pension plan. Universal would not be considered a subsidiary because it is only 55% not more than 80%. The union employees are excluded from testing by the IRC.

152
Q

Which of the following transactions by a qualified plan’s trust are subject to Unrelated Business Taxable Income (UBTI)?

I. A trust obtains a low interest loan from an insurance policy it owns and reinvests the proceeds in a CD paying a higher rate of interest.
II. A trust buys an apartment complex and receives rent from the tenants.
III. The trust buys vending machines and locates them on the employer’s premises.
IV. The trust rents raw land it owns to an oil & gas developer.

I and II only.
I and III only.
II and IV only.
I, II and IV only.

A

I and III only.
- Statements “I” and “III” are subject to UBTI because income from any type of leverage or borrowing within a plan is subject to UBTI. Additionally, any business enterprise run by a qualified plan is subject to UBTI. Statement “II” is not subject to UBTI (assuming it is not subject to leverage) due to a statutory exemption for rental income. Statement “IV” - The rental of raw land is also exempt. If the plan actually participated in the development of the oil & gas reserves, there would be UBTI.

153
Q

Which of the following statements are accurate concerning integration (“permissible disparity”) rules for qualified plans?

I. The integration base level for a defined contribution plan can exceed the current year’s Social Security taxable wage base.
II. Permitted disparity levels reduce benefits in a defined benefit plan if employee retires early.
III. It isn’t possible to have a defined benefit plan formula which eliminates benefits for lower paid employees.

I only.
I and II only.
II and III only.
I and III only.

A

II and III only.
- Statement “I” is incorrect because the integration levels cannot be higher than the Social Security wage taxable wage base. It may be lower, but cannot be higher. All other statements are accurate

154
Q

Mike’s Mega Muffelettas (MMM) is a fairly large company based in Louisiana, with over 300 employees. MMM sponsors a defined benefit plan. George has worked at the company for the last 30 years and is looking forward to his retirement in another ten years. However, he just received a letter from the company that informs him that his defined benefit plan is being converted to a cash balance plan. What advice can you give George?

His benefit could freeze as a result of the conversion; a stituation known as “wearaway.”
The present value of the accrued benefit from the defined benefit should be preserved in the conversion and you should earn additional benefits under the cash balance plan.
The cash balance plan provides a guaranteed rate of return and benefits that are fully insured by the PBGC.
The cash balance plan acts like a defined contribution plan and will permit Geoge to self-direct his retirement assets.

A

The present value of the accrued benefit from the defined benefit should be preserved in the conversion and you should earn additional benefits under the cash balance plan.
- Answer b is correct because his benefits under the defined benefit plan must be preserved. Answer a is not correct as “wearaway” was done away with as it negatively effected employees who were near retirement and resulting in employees not accruing additional benefits after a conversion from a defined benefit plan to a cash balance plan. Answer c is not correct as the PBGC does not fully guarantee benefits. Answer d is not correct as a cash balance plan will have a guaranteed rate of return. Employees do not self-direct their assets in a cash balance plan.

155
Q

Which one of the following is a possible disadvantage of a Simplified Employee Pension plan (SEP) for an employer?

The SEP’s trustee is subject to ERISA’s prohibited transaction excise tax penalties.
A SEP must have a fixed contribution formula that is non-discriminatory.
SEPs prohibit forfeitures.
Employer contributions to a SEP are subject to payroll taxes.

A

SEPs prohibit forfeitures.
- Options “A” and “D” are false. Option “B” is a mandatory characteristic of all qualified defined contribution pension plans but not profit sharing plans.

156
Q

SEP-IRA plans are unique from defined contribution plans in which of the following areas:

I. Length of permissible exclusion from coverage based upon service.
II. Establishment date of the plan.
III. Income requirements for participation.
IV. Can be paired with another plan.

I, II and IV only.
I, II and III only.
I, II, III and IV.
None of the above.

A

I, II and III only.
- Employees can be excluded up to 3 years or age 21, whichever is longer. Plans can be established and funded up to the date of filing the entity tax return, including extensions. Employee needs to earn only $600 to be included in the plan.

157
Q

Which of the following accurately describes the similarities between a traditional Individual Retirement Account and a SEP-IRA?

I. Individual ownership of the account.
II. All contributions into the account are fully owned by participant.
III. Subject to early withdrawal penalties and minimum distribution regulations.
IV. All distributions from plan taxed as ordinary income.

I and III only.
II and IV only.
II, III and IV only.
I, II, III and IV.

A

I, II, III and IV.
- All of these characteristics are shared by traditional IRAs and SEP-IRAs. Any non-deductible contribution to an IRA are taxed as a pro-rata distribution.

158
Q

A small business owner in a very specialized field, establishes a SEP for his proprietorship after 2008. He has two employees of 2 1/2 years making $80,000 per year. He uses the statutory maximum exclusions for all employees. He earns $100,000 in modified earned income. What is the maximum allowable plan contribution to the owner’s account?

$16,000
$20,000
$25,000
$40,000

A

$20,000
- Maximum contribution into a SEP is 25% or $20,000 per employee. The modification for the owner, a self-employed person is, would be: EE contribution % / 1 + EE contribution %. In this case the calculation would be .25 / 1.25 = .2; $100,000 × .2 = $20,000 maximum contribution for the business owner.

159
Q

Kent Reeder, age 52, works as the administrator and curator at the Museum of Antique Manuscripts, a not-for-profit organization in Metropolitan Center. He has worked there 18 years and began contributing to the 403(b) plan 12 years ago but skipped contributing last year. He earns $85,000 a year. He has asked you to maximize his contribution. Which of the following is/are TRUE?

I. He may contribute $19,000 plus $6,000 for age 50+ catch-up, plus $3,000 long service catch-up.
II. He may not contribute to the long-service catch-up this year due to omitting a contribution last year.
III. He may contribute $19,000 plus $6,000 age 50+ catch-up.
IV. He may not participate in both the long service catch-up and the age 50+ catchup the same year.
V. He is not eligible for the long service catch-up.

III and V only.
II only.
I, III and IV only.
I and III only.

A

III and V only.
- He is not eligible for the long service catch-up because the museum is not a Health, Education, Religious (HER) organization. The maximum contribution limits for 2019 are $19,000 plus the age 50+ catch-up of $6,000.

160
Q

Which of the following statements concerning rabbi trusts is (are) CORRECT?

I. A rabbi trust is a trust established and sometimes funded by the employer that is subject to the claims of the employer’s creditors, but any funds in the trust cannot generally be used by or revert back to the employer.
II. A rabbi trust calls for an irrevocable contribution from the employer to finance benefits promised under a nonqualified plan, and funds held within the trust cannot be reached by the employer’s creditors.
III. A rabbi trust may not be held off-shore as a result of the American Jobs Creation Act of 2004.
IV. The American Jobs Creation Act of 2004 prohibits “springing irrevocability” for a rabbi trust if there is a change of control or ownership.

I and IV only.
I and III only.
II and III only.
I only.

A

I and III only.
- Statement “II” describes a secular trust. Statement “IV” is incorrect because AJCA 2004 does allow springing irrevocability in these circumstances, but not for bankruptcy.

161
Q

Harry wants to retire at age 62 in the current year. To be eligible for reduced OASDHI retirement benefits, how many quarters of coverage must Harry have earned?

6
13
20
40

A

40

- Harry must have earned 40 quarters of coverage.

162
Q

Which of the following would be eligible to receive Social Security retirement benefits (assume each has worked at least a minimum of 20 years in their current position)?

I. A 61-year old teacher at a public school.
II. A 60-year old 25% owner of an S corporation.
III. A 65-year old physician/owner of a professional corporation.
IV. A 70-year old sole proprietor who is an outside consultant to the federal government.

III and IV only.
I, III and IV only.
IV only.
I, II, III, and IV.

A

III and IV only.

- Persons I and II are too young and do not qualify. The earliest age for social security retirement benefits is 62.

163
Q

Which of the following statements concerning the OASDHI earnings test for the current year is correct?

Some part-time work is allowed without the loss of retirement benefits for those under normal age retirement.
The earnings test does not apply after the age of 62.
Interest and dividends are included in the earnings test.
The annual exempt amount for a person at normal age retirement is $46,920.

A

Some part-time work is allowed without the loss of retirement benefits for those under normal age retirement.
- The earnings test does not apply at, or after, normal age retirement. The monthly exempt amount is $3,910 ($46,920) in 2019 for those months in the year of normal retirement age BEFORE you actually reach normal retirement age. The test uses only earned income. No passive or portfolio income is used in calculating the earnings.